Jump to content

Wikipedia:Reference desk/Science

From Wikipedia, the free encyclopedia

This is an old revision of this page, as edited by Keegstr (talk | contribs) at 15:08, 30 June 2011 (→‎What is this little white bug?). The present address (URL) is a permanent link to this revision, which may differ significantly from the current revision.

Welcome to the science section
of the Wikipedia reference desk.
Select a section:
Want a faster answer?

Main page: Help searching Wikipedia

   

How can I get my question answered?

  • Select the section of the desk that best fits the general topic of your question (see the navigation column to the right).
  • Post your question to only one section, providing a short header that gives the topic of your question.
  • Type '~~~~' (that is, four tilde characters) at the end – this signs and dates your contribution so we know who wrote what and when.
  • Don't post personal contact information – it will be removed. Any answers will be provided here.
  • Please be as specific as possible, and include all relevant context – the usefulness of answers may depend on the context.
  • Note:
    • We don't answer (and may remove) questions that require medical diagnosis or legal advice.
    • We don't answer requests for opinions, predictions or debate.
    • We don't do your homework for you, though we'll help you past the stuck point.
    • We don't conduct original research or provide a free source of ideas, but we'll help you find information you need.



How do I answer a question?

Main page: Wikipedia:Reference desk/Guidelines

  • The best answers address the question directly, and back up facts with wikilinks and links to sources. Do not edit others' comments and do not give any medical or legal advice.
See also:


June 26

Hotspot underneath Iceland is the same one that produced the Siberian Traps?

In this article, it is mentioned that the hotspot that produced the Siberian Traps may be underneath Iceland now. But do have have 250 million years of continuous volcanism from today in iceland to the formation of the Siberian Traps, 250 million years ago? Count Iblis (talk) 00:46, 26 June 2011 (UTC)[reply]

The article you cited did not claim that the plume that caused the Siberian trapps is now under Iceland. It said that a gigantic impact in Antarctica may have caused the Sib. trapp by antipodal disruption and incidentally may also have contributed to the Iceland plume. 190.148.136.161 (talk) 01:49, 26 June 2011 (UTC)[reply]

Prehistoric Scotland says that there was a lot of volcanic activity around the Scotland area in that same time period. The Icelandic plume is thought to have migrated N-W. from that area helping to open the North Atlantic.190.148.136.161 (talk) 02:07, 26 June 2011 (UTC)[reply]

It's a bit of a jump to get from the Siberian Traps to the Iceland plume, although in between there is the High Arctic LIP, which was mainly active in the middle to late Cretaceous (so about 100 to 80 million years ago), and the North Atlantic LIP which was active from the latest Cretaceous up to the end of the Paleocene (about 70 to 55 mya). The iceland plume reached its current location in the Mid-Atlantic Ridge by the Miocene (about 15 mya). Some people have suggested [1] following plate reconstruction that the siberian traps are located exactly where the North Atlantic LIP was to erupt some 180 my later, but I'm not sure that this is the view of everyone working on these things. Mikenorton (talk) 18:14, 26 June 2011 (UTC)[reply]
thanks! Count Iblis (talk) 15:13, 27 June 2011 (UTC)[reply]

crossing over

If I stood, or sat in my rowboat, at the east side of the international date line (somewhere in the Pacific), and then I stepped or rowed to the other side of the line. Would today now be tommorrow or yesterday? 190.148.136.161 (talk) 00:59, 26 June 2011 (UTC)[reply]

If it is midday on Tuesday on the east side of the line then it is midday on Wednesday on the west side of the line. See International Date Line for more information. --Tango (talk) 01:20, 26 June 2011 (UTC)[reply]
Of course, today is still today on either side of the line. It is simply that our time-keeping convention is to call today Tuesday on one side of the line, and to call today Wednesday on the other side of the line. It's just an arbitrary change of co-ordinates. Gandalf61 (talk) 08:12, 26 June 2011 (UTC)[reply]
It affects business in the South Pacific. When it is Monday morning in New Zealand and Australia it is Sunday morning on some Pacific Islands and the businesses there then suffer. Some Pacific Islands have decided to move one day ahead to deal with this problem. Count Iblis (talk) 17:07, 26 June 2011 (UTC)[reply]
The way to remember which is which is always to remember that as land moves to the east it moves later in time. (i.e. the Sun rises up over it) So the west edge of the date line is the furthest point east in the world, and so it's the latest time zone.
My suspicion is that there are many small and large ways (also road conventions, negotiating teams, human rights issues, Whale Wars) in which it looks like Australia is becoming more of a regional power over much of the South Pacific. Wnt (talk) 04:21, 27 June 2011 (UTC)[reply]

Constitutional walk [close to speed of light] on Einstein’s treadmill:

Although the stepping positions in space would remain same but would the moving clock on such a treadmill be slowed down? As air time is involved in RUNNING therefore WALK should be considered in the scenario - non-broken connection between walker and walk belt 68.147.41.231 (talk) 03:21, 26 June 2011 (UTC) Eccentric Khattak No.1[reply]

Yes, a clock attached to the treadmill belt would run slow, as observed in the racewalker's inertial frame of reference.
The footfalls all occur at the same location in space according to the racewalker's frame of reference (or really the same two locations, since the racewalker has two feet), but they do not all occur at the same location according to the belt's frame of reference. In the belt's frame of reference, the footfalls occur at the locations of the footprints left behind on the belt, which in the belt's frame of reference are stationary.
The two frames of reference do not necessarily agree as to whether the person is running or walking. The event of a front foot landing and the event at around the same time of a rear foot leaving the belt occur at two different locations, so there's relativity of simultaneity to deal with. I.e., the two frames of reference may disagree as to whether the rear foot has left the belt yet as of when the front foot lands. Red Act (talk) 12:06, 26 June 2011 (UTC)[reply]
Also, unless the belt is moving less than half the speed of light (in the racewalker's frame of reference), walking instead of running would be impossible, because walking would require the feet to move faster than the speed of light. Red Act (talk) 12:41, 26 June 2011 (UTC)[reply]
The person can be walking in their frame no matter how fast the treadmill is going, it just won't necessarily be walking in the treadmill frame. Rckrone (talk) 16:03, 26 June 2011 (UTC)[reply]
How do you define "their frame" for someone walking? Is that the frame of their head? I don't think it is possible to walk (on solid ground or a treadmill) faster than around 0.5c, since your feet need to move faster than your overall speed (in the lab frame) in a walking motion. The same is true of a vehicle travelling on wheels - the top of the wheel moves at twice the speed of the vehicle, so it can't go faster than 0.5c (in that case, it is exactly 0.5c, I'm not sure exactly what the top speed would be for a walker). --Tango (talk) 18:30, 26 June 2011 (UTC)[reply]
Yeah, I do mean their head. That seems like the only reasonable way to define "their frame". In that frame there's clearly no problem with having one leg moving at say 3c/4 in one direction and the other leg moving at 3c/4 in the other, and then at some regular interval both quickly switching direction (if you ignore the difficulty of accelerating a leg so much). Here I'm imagining the body and two legs as 3 separate objects. From the lab frame the person would be running, since the lift of the back foot would be well before the fall of the front one. The two leg speeds would be 0 and 24c/25, but clearly each leg would be spending the majority of the time moving forward since the average speed is still 3c/4. I can't think of any reason why c/2 would be any more of a barrier with wheels, but things get messy trying to describe "rigid" bodies rotating like that. Rckrone (talk) 05:07, 27 June 2011 (UTC)[reply]
In the lab frame, if a car is moving at c/2 then the top of its wheels are moving at c, hence the car cannot move at c/2 or faster (in reality, if you tried the wheels would spin - what we've found is a fundamental limit to friction between the wheel and the road). --Tango (talk) 11:52, 27 June 2011 (UTC)[reply]
In the frame of the wheeled vehicle, all we have is a rotating object that has a speed of c/2 at the edge. There's nothing impossible about that (besides holding an object together under so much force). In the lab frame the top of the wheel is not moving at a speed of c, it's going at 4c/5. From this perspective the wheel is pretty distorted, seeing as more of the outside edge of the wheel is on the top than on the bottom at any given time. You can think of the wheel case is basically like having a lot of feet. Rckrone (talk) 15:43, 27 June 2011 (UTC)[reply]
Yeah, that was another poorly worded sentence on my part. I should have put "(in the treadmill belt's frame of reference)" after the phrase "walking instead of running" in my post, because otherwise it sounds like the racewalker's frame of reference is being used to define walking vs. running, since that's the only frame of reference mentioned in that sentence.
I'll spell out and explain the c/2 limit in detail to make it clear to everybody (hopefully). In any reference frame used to define "running" vs. "walking", assuming the left and right footfalls are evenly spaced in terms of time and distance, "walking" means that each foot spends at least 50% of the time (in that reference frame) in contact with the treadmill belt, so that the other foot is in contact with the belt both when the given foot lands and leaves the belt. In the treadmill belt's frame of reference, let t1 and t2 be two consecutive times at which the left foot lands, and let x1 and x2 be the corresponding locations at which the left foot lands. If the belt is moving at speed v in the lab frame, then in the treadmill belt's frame, the person is moving at speed v, which implies that (x2-x1)/(t2-t1) = v. For the racewalker to be walking in the treadmill belt's frame of reference, the earliest the left foot can leave the belt during that cycle is at t1+(t2-t1)/2. That means that during the second part of that cycle, the left foot only has at most a duration of (t2-t1)/2 in which to cover a distance of x2-x1, meaning its average velocity during that second part of the cycle must be at least (x2-x1)/[(t2-t1)/2] = 2(x2-x1)/(t2-t1) = 2v, which would be superluminal if v > c/2. Red Act (talk) 07:06, 27 June 2011 (UTC)[reply]
A clock attached to the treadmill would obviously run slow relative to a clock on the wall of the room (which I assume is the reference frame the OP is interested it), the fact that someone is running on it makes no difference at all. I don't think that was the question, though. I think the question was whether a clock attached to the runner would run slow compared to the wall clock. If it was attached to the runner's torso, then it wouldn't since the torso isn't moving significantly relative to the wall (that lots of other things are moving really quickly doesn't matter, you just look at the two clocks). If it were attached to one of the runner's legs, then it gets a lot more complicated because the legs are accelerating. At some points they will be moving backwards at relativistic speeds, at some points they will be stationary and at some points they will be moving forwards at relativistic speeds. That means the clock on the legs will run slow compared to the wall clock, but I can't immediately calculate by how much. --Tango (talk) 13:34, 26 June 2011 (UTC)[reply]
The other replies have effectively already said this, but let me emphasize: time dilation only depends on the motion of the clock relative to the lab. It never makes any difference how the clock is "attached" to the lab (via a pair of rapidly moving legs and a treadmill, in this case). Likewise, it makes no difference if it's not attached at all for some of the time (air time). In general relativity, the legs and the treadmill do technically matter, but only because they gravitate, not because they're attached to the clock. -- BenRG (talk) 16:47, 26 June 2011 (UTC)[reply]

To all - Sorry about the confusion!

By “Moving Clock”, I meant light clock [mini model] on the Runner’s head [fixed] not attached to the walk belt. As runner [walker] moves close to the speed of light and so the light clock therefore if time dilates then

Why a pulse doesn’t trace out longer path/ angled for stationary observer in Gym’s frame of reference?

Why same vertical bouncing of pulse between two mirrors for both moving [runner] and stationary observers? 68.147.41.231 (talk) 03:07, 27 June 2011 (UTC) Eccentric Khattak No.1[reply]

Let make it more simple – Let a person stands [on his Roll Skates] on aforementioned treadmill. Although a person seems stands still but the walk belt enable his wheels to roll on its smooth surface. 68.147.41.231 (talk)EEK No.1 —Preceding undated comment added 03:58, 27 June 2011 (UTC).[reply]

Just because a person is rolling along on a treadmill doesn't mean that he's moving. When you talk about something moving, you always have to specify what it is that the movement is relative to. In this case, the person, the treadmill, and the lab the treadmill is in all have the same comoving frame, i.e., none of them are moving relative to each other. In that lab frame, only the treadmill belt is moving (and the roller skate wheels). So as measured in the lab frame, the clock attached to the person's head does not exhibit time dilation, because its speed as measured in the lab frame is zero. However, that clock attached to the person's head would be running slow as measured in the belt's comoving frame, and a clock that moves along with the belt runs slow as measured in the lab frame. Red Act (talk) 04:24, 27 June 2011 (UTC)[reply]
If you're running on a treadmill, the only thing moving is the treadmill. And the treadmill is moving back and forth, just like the spaceship in the twins paradox, so it will end up with a different span of time like the accelerated twin. Wnt (talk) 04:26, 27 June 2011 (UTC)[reply]
When I said the treadmill wasn't moving relative to the lab frame, I meant the non-belt portion of the treadmill, not the belt. The belt is obviously moving relative to the lab frame, and moving relative to the non-belt part of the treadmill. I probably should have worded things differently. Red Act (talk) 04:36, 27 June 2011 (UTC)[reply]

"9,900 years hyperbolic orbit"?

At http://elenin.org/ we read that this comet "follows a 9,900 years hyperbolic orbit around our sun". That seems contradictory. To specify a number of years seems to imply periodicity. But "hyperbolic" seems to imply that it's not periodic. What is meant? Michael Hardy (talk) 04:34, 26 June 2011 (UTC)[reply]

At one Epoch the comet can be hyperbolic, while at another it can be a closed loop. The real orbit (or the best approximation to such) considers perturbations by all planets, a few of the larger asteroids, a few other physical usually small forces, and requires numerical integration. Comet Elenin's orbit will become closed because Jupiter's gravity will lower the eccentricity below 1 as Elenin is leaving the planetary region. -- Kheider (talk) 05:24, 26 June 2011 (UTC)[reply]
Confusion between orbit types is not the most surprising thing at http://elenin.org/. Further down the page, it speculates that Elenin is in fact a brown dwarf and that it will "preciptate major reactions within the Earth's core as well as on its surface that could very well lead to a global catastrophe". Apparently it is connected with the Nibiru collision theory and the whole 2012 "end of the world" thing. Gandalf61 (talk) 08:06, 26 June 2011 (UTC)[reply]

It is often said that Adamantane is the smallest molecule that technically, can be called a diamond. Is it really so, since it contains no carbon only moeities? I suggest a new basic diamondoid (SMILES): C1C2CC3CC1C14C5CC6CC(C5)C57C8CC9CC(C8)C3(C3CC1CC5C3)C47C269. A similiar case exists for Lonsdaleite, where its basic unit is bicyclo[2.2.2]octane. New basic Londsdaloid: C1C2CC3CC4CC5CC6CC7CC8CC9CC1C81CC8CC24C1(C57C8)C369. What are the major differences besides their size and mass? Plasmic Physics (talk) 04:41, 26 June 2011 (UTC)[reply]

I guess it depends on what you think the essential nature of diamond is. Is it being just carbon, or is it being a lattice of singly-bonded tetrahedral carbons? If you're single bonded tetrahedral carbon, you're going to have edge effects, where you cap the carbon with something else (like hydrogen). The question is how far in you allow the edge effects to penetrate. For example, for Adamantane, it's all edge. I would say that Adamatane *does* have a carbon-only moeity, which consists of the ten carbons bonded together. Yes, it's directly connected to hydrogens, but why should that matter? If you're just looking for an all-carbon moeity with tetrahedral structure, why doesn't neopentane count? Or 1-methyladamantane? -- 174.24.222.200 (talk) 18:20, 26 June 2011 (UTC)[reply]

Well, I didn't choose them, because neopentane is non-reprasentative of a singular allotrope; and I chose the ones I did because the central carbon forms closed hexagons with all four adjacent atoms, meaning at it contains five carbons which are only attatched to other carbons. That is what I meant by carbon only moeities, appologies for the confusion. Plasmic Physics (talk) 20:35, 26 June 2011 (UTC)[reply]

The simplest carbon-only moeties I can think of would be the fullerenes, but they have graphite-type bonding rather than diamond-type bonding. The problem is one of geometry; graphite- type bonding, with its planar 120 degree bonding, is relatively easy to bend back on itself and leave no open branches. With diamond-type bonding, being tetrahedral (three dimensional and 109.5 degrees), tends to create more branches the further and further you get from the "origin" as you branch out, it becomes impossible to cap it with only more carbons. Eventually, you need to cap it with some univalent atom to end the branching, so even a large diamond crystal (which is essentially a single molecule) has all of its branches terminated by hydrogen. If you are dealing with closed hexagons, you're back to fullerenes and graphite. --Jayron32 18:40, 27 June 2011 (UTC)[reply]

Yes, but only the central carbon forms closed hexagons in my examples. That is the key to my structures. According to my theory, the basic fullerene should be 1-dehydrophenalene: c1cc2cccc3cccc(c1)c23. Plasmic Physics (talk) 11:23, 30 June 2011 (UTC)[reply]

what is un-saturated polyester film

Dear Sir,

As per subject given up Kindly clearify my that what is diffrence between saturated and un-saturated polyester film . This question is asked because of in international harmonized code 3920-6310 of about polyester film which film shall applied for this code its not clear as fo as you are requested to please clerifiy me about diffrence between saturated and un-sturated polyester film and for which purpose these film shall be used.

Thanks and best regards usman hafeez -redacted-

LOL @ "un-sturated". StuRat (talk) 02:32, 27 June 2011 (UTC) [reply]
See Polyester and Polyester resin. Unsaturated polyesters are resins, generally used in structural applications (as "fibreglass"). I don't think it would be possible to make a _film_ from resin, so anything described as "polyester film" will almost certainly be made of a saturated polyester. Tevildo (talk) 11:27, 26 June 2011 (UTC)[reply]

Orbital speed and escape velocity

My book on physics says: "The higher the [circular] orbit of a satellite, the slower its speed and the longer its period." But it doesn't say anything about why this is so. My reasoning is that the escape speed farther from the earth is lower, so the speed of the satellite also needs to be lower, otherwise it won't stay in its orbit but escape. Is my reasoning correct? Lova Falk talk 09:50, 26 June 2011 (UTC)[reply]

Yes, it is. Plasmic Physics (talk) 10:21, 26 June 2011 (UTC)[reply]
Sort of. A better way to look at it is in terms of centripetal force. The centripetal force is the force that keeps the object on a circular path, in this case that is gravity. That article gives us a formula for centripetal force: F=mv2/r (r is distance from the centre of the orbit, m is the mass of the object and v is its velocity). If you equate that to Newton's Universal Law of Gravity, F=GMm/r2 (M is the mass of whatever it is orbiting and G is the gravitational constant) and rearrange, you'll get v2=GM/r. That shows that as r increases, v decreases. That the orbital period will increase is obvious - it's travelling a longer distance at a slower speed, so it takes longer. --Tango (talk) 14:35, 26 June 2011 (UTC)[reply]

Looking for a good Scientific Paper

about the PRINCIPLES of the Imprinting phenomenon of Konrad Lorenz.

Many thanks. — Preceding unsigned comment added by 79.179.8.59 (talk) 12:45, 26 June 2011 (UTC)[reply]

I'm not sure what "principles" means, but in any case Google Scholar is often a good resource for digging up information of that sort. You can find a downloadable pdf of a 1958 Scientific American article here; or if you are looking for something more recent, this is a paper from 2011 that briefly describes the current state of the art. There is lots more available; the literature on that phenomenon is very extensive. Looie496 (talk) 17:11, 26 June 2011 (UTC)[reply]

Subterranean rivers

The concept of subterranean rivers somehow makes me feel as if I'm thinking about strange creatures from mythology, and I wonder if that's because I've found rather little information about them. Here we see one of a number of places in the Minneapolis/St. Paul area where such a stream emerges from underground at a point along the banks of the Mississippi River; at each such point there's a waterfall because the riverbanks in that vicinity are steep and high. From this map it looks as if this stream must flow under the campus of the University of St. Thomas. Obviously the locations of such things must be taken into account whenever a building is built on the land above it.

Where can I find maps of subterranean rivers in specified areas (Google Maps doesn't immediately give me those).

Leonardo da Vinci wrote that subterranean rivers flow from the Caspian Sea to the Black Sea. Is there any truth in that? Michael Hardy (talk) 19:38, 26 June 2011 (UTC)[reply]

If the mental picture you have is of an empty tube, filled with water, like a pipe or tunnel, that's extremely rare. Groundwater emerging from the terrain is almost always the outflow from an aquifer. Very occasionally, in the right circumstances, there can be flooded caves, but that's very rare. Anyone planning any building of reasonable size will perform a geotechnical survey which will determine the technical nature of the soil and the character of the water table (and other local hydrology) and will design the building's foundation accordingly. -- Finlay McWalterTalk 19:49, 26 June 2011 (UTC)[reply]
In cities it's not uncommon for natural rivers to be put into culverts and then built over, such that they become effectively subterranean rivers (see Subterranean rivers of London for some examples). It's mostly a matter of nomenclature whether these are really rivers or just drains. -- Finlay McWalterTalk 20:01, 26 June 2011 (UTC)[reply]
@Finlay: I don't think you would suspect that the stream in the map I linked to had been artificially put underground if you saw it close up.
Here's another one, maybe a mile and a half upstream from there. The stream flows out of the vertical face of a cliff maybe 30 feet below street level, and looking around you would see that the waterfall has been carved out over many centuries. Michael Hardy (talk) 22:11, 26 June 2011 (UTC)[reply]
Yes, but it could still be water flowing out of porous rocks, as opposed to a tube. If you fill a keg with sponges and water, then open the tap, water will flow out, but have you created a "river" ? StuRat (talk) 02:29, 27 June 2011 (UTC)[reply]
So if I want to find out whether a particular place where flowing water emerges from the ground is from a hollow channel or from porous rock, are there standard maps that I can find on the internet or in libraries that indicate that? Michael Hardy (talk) 20:34, 27 June 2011 (UTC)[reply]
The article on cave diving describes formations that are more river-ish in Florida and Yucatan (the first being karst limestone, the second cenotes where limestone has been impacted by the asteroid that killed the dinosaurs) I'm sure there must be many other good examples. Wnt (talk) 04:12, 27 June 2011 (UTC)[reply]
According to the internet one example of an underground river is in Battlefield Cavern, part of White Scar Caves in England. 92.29.127.234 (talk) 09:17, 27 June 2011 (UTC)[reply]
This site claims that a crypto-river has been tracked underneath the Nile. --TammyMoet (talk) 12:39, 27 June 2011 (UTC)[reply]

If people want to cite examples, there's Mammoth Cave in Kentucky, which IIRC was carved by a river flowing through it, under ground through most of its course. It's a tributary of the Ohio River. And in Bellefonte, Pennsylvania, a river emerges from the ground, which flows underground until it reaches that point and above ground downstream from there. Michael Hardy (talk) 17:32, 27 June 2011 (UTC)[reply]

Something else that relates is rivers that flow part-time. You may have a regular above-ground river at times, then perhaps it drops below the surface but continues to flow at other times, then stops entirely in the driest periods. Dotted lines are often used to show such intermittent rivers. StuRat (talk) 20:40, 27 June 2011 (UTC)[reply]

what is older?

A meteorite or gold? Or... were they formed at the same time? — Preceding unsigned comment added by 92.30.142.163 (talk) 21:16, 26 June 2011 (UTC)[reply]

Gold is a chemical element, so it will have been created in a supernova and will have been essentially unchanged ever since (gold rarely even bonds with other elements in compounds). Some of it may have come from the radioactive decay of other elements, but I believe that is a very small proportion. Meteorites, on the other hand, are lots of compounds stuck together. Meteorites will have formed during the early period of the formation of the solar system, which was some time (ie. millions, maybe billions of years) after the supernova that seeded the solar nebula with heavy elements. --Tango (talk) 21:27, 26 June 2011 (UTC)[reply]
(edit conflict)Your question really is asking to compare apples to oranges. A meteorite is defined as any extraterrestrial object, whether rock or metal, that has fallen to the Earth's surface. These have been falling to Earth since its formation, and many of the meteorites that fall are remnants from the early solar system before the earth was formed, but technically they are not meteorites until they land on Earth.
Gold, on the other hand, is an element; it is formed in the heaviest stars as they turn to supernovae. Gold we find on Earth was formed as stars exploded over and over during the billions of years before the formation of our solar system. Thus, in the strictest definition, you could say that gold is "older" than meteorites, but again, it is not really a good question to ask. Some meteorites may even contain gold. -RunningOnBrains(talk) 21:36, 26 June 2011 (UTC)[reply]
I'd say almost all gold on Earth is older than meteorites on Earth, having been created by supernovae before the formation of our solar system (and thus before any meteorites here), but there could be some more recently created gold that drifted into our solar system and fell to Earth since it formed, and therefore could possibly be newer than some terrestrial meteorites. Also, gold can be created as the end product of radioactive decay, so there might be some newer gold from that source. As for the broader question of gold versus meteorites throughout the universe, that's a different story. I would think there were many meteorites prior to the first supernova, so they might win, in that case. StuRat (talk) 02:21, 27 June 2011 (UTC)[reply]
Meteorites need elements heavier than helium in order to form, so they must have come after the deaths of the first stars. The first stars are predicted to have been really large and would have gone supernova, producing gold. See Population III. Therefore gold existed in the universe before meteorites did (well, before meteors did - they weren't meteorites until there were planets from them to crash into!). --Tango (talk) 12:48, 27 June 2011 (UTC)[reply]
If the questioner is wondering about specific formations of gold, e.g. nuggets, veins, etc. Then it is worth noting that these are usually relatively recent geologic formations (e.g. 10s or 100s of million of years ago). The geologic processes that give rise to mineral veins are still ongoing, so a gold deposit could be of virtually any age. As discussed above, the gold atoms where created in supernova before the birth of our solar system, but specific gold deposits are likely to have been put in place much more recently. Dragons flight (talk) 17:46, 27 June 2011 (UTC)[reply]
Some meteorites are potentially over 10 billion years old. Solid gold requires it being expelled from Earth's interior. ~AH1 (discuss!) 01:35, 30 June 2011 (UTC)[reply]

Asperger's syndrome and stalking

Are there any empirical studies relating Asperger's syndrome and stalking? And what are the results..? --helohe (talk) 23:55, 26 June 2011 (UTC)[reply]

I don't know if there are, but I can tell you that I have definitely never stalked or considered doing so to anyone. Plasmic Physics (talk) 00:00, 27 June 2011 (UTC)[reply]
But, are you an aspie? — Preceding unsigned comment added by Wikiweek (talkcontribs) 13:17, 27 June 2011 (UTC)[reply]
Yes, I am. Plasmic Physics (talk) 11:26, 30 June 2011 (UTC)[reply]
Yes, this downloadable paper from 2007 directly addresses that question, and its introduction reviews earlier relevant literature. The results "show that the diagnosis of ASD is pertinent when individuals are prosecuted under stalking legislation in various jurisdictions". I will leave it to you to figure out what that means. Looie496 (talk) 02:06, 27 June 2011 (UTC)[reply]
I think the previous sentence from that paper is an even better one to quote here: "Individuals with [autistic spectrum disorders] were more likely to engage in inappropriate courting behaviours..., and were more likely to focus their attention upon celebrities, strangers, colleagues, and ex-partners ..., and to pursue their target longer than controls." Red Act (talk) 04:05, 27 June 2011 (UTC)[reply]


June 27

What is the biggest Humpback whale ever found?

What is the biggest humpback whale ever found? Neptunekh2 (talk) 00:46, 27 June 2011 (UTC)[reply]

According to this, "maximum reliably recorded adult lengths are in the 16-17 meter range". ~ Mesoderm (talk) 01:29, 27 June 2011 (UTC)[reply]

life and reproduction

Would it be scientifically accurate to say that by having a child, I am perpetuating 3.5 billion years of successful reproduction, and therefore extending a 3.5 billion year long lineage by one generation? The Masked Booby (talk) 02:49, 27 June 2011 (UTC)[reply]

That's probably roughly right, although the lineage conceivably might have been a lot longer than that if life on Earth started here via exogenesis. Note that in any event, exactly how many generations it took to create you isn't precisely defined, because sexual reproduction can mix the genes of individuals of widely different ages. There's presumably also a little vagueness in the generation count due to it being a little vague as to when during the process of abiogenesis there began existing clearly defined cells, with clearly defined parent cells. However, the vast majority of generations of your ancestors since life began involved asexual reproduction of single-celled organisms, so the vagueness in the number of generations wouldn't be that large of a percentage. Red Act (talk) 03:57, 27 June 2011 (UTC)[reply]
The simple answer is "Yes". Now, if the OP is trying to figure out how many generations that translates to, it gets slipperier. Just among my own 16 great-great-grandparents, there is 37-year gap from oldest to youngest. That's more than the average length of a "generation" (20-30 years) just by itself. ←Baseball Bugs What's up, Doc? carrots04:24, 27 June 2011 (UTC)[reply]
Actually more than one: your mitochondria have their own proud parentage. But if we credit that, we should also consider horizontal gene transfer which, in a sense, means that we could have parentage through many small bits of many different microbes in the billions of years past - before at last converging on some common ancestor(s) of all life in yet earlier generations. Wnt (talk) 04:30, 27 June 2011 (UTC)[reply]
A microbe called Adam? :) ←Baseball Bugs What's up, Doc? carrots04:43, 27 June 2011 (UTC)[reply]
Don't forget Y-chromosome DNA. ~AH1 (discuss!) 01:33, 30 June 2011 (UTC)[reply]
I get the impression from the above discussion that a great many of my ancestors were probably single-celled organisms and thus incapable of creating written genealogical records. If so, this explains why I have been unsuccessful in tracing my family tree back further than about 10,000 years (as well as other issues I have puzzled over). This is a huge relief! Thank you very much. Wanderer57 (talk) 20:16, 2 July 2011 (UTC)[reply]

Rock-hard stools and fake fruit juice

As a child I learned by experience that drinking too much fake fruit juice led to terrible stools so rock-hard I could only pass them by pulling them out piece by piece with my fingers. So I've long since avoided anything but real fruit juice - though on two or three occasions at hotels or social events I've forgotten this precept until unpleasantly reminded that it still applies even as an adult. But looking over the web it seems like fruit juice is commonly recommended to people with fecal impaction (which sounds like a more serious version of this), and it seems like advertisements for stool softeners have grown very common in the U.S. in recent years. (the effect of home budget cutbacks...?) So in the interest of all the kids who haven't learned by trial and error yet I'm curious - can someone think of evidence that might link this problem to some ingredient in fake fruit juice? The first guess that comes to mind is high fructose corn syrup... yet I get exposed to so much of that in other forms without any similar effect ever happening. Unfortunately, because of when and how I was exposed, I don't know the ingredients of the products involved, though one of the worst culprits vaguely resembled orange juice. I couldn't find anything on PubMed in a few basic searches... but I'm not sure what I'm looking for. Wnt (talk) 05:56, 27 June 2011 (UTC)[reply]

For making that discovery I'd like to shake your hand ... or maybe not. :-) StuRat (talk) 06:23, 27 June 2011 (UTC)[reply]
But, seriously, if everyone who drank such drinks had that reaction, it would be widely known already. You must have had an allergy or unusual reaction. Also, I would bet that parents who give their kids fake juice also don't tend to give them enough fiber and provide a generally poor diet, overall, so any of those factors could contribute. StuRat (talk) 06:23, 27 June 2011 (UTC)[reply]
I don't see anything on PubMed about allergy causing fecal impaction. While it may not be a universal reaction I somehow doubt it's all that rare, judging by the prevalence of hard stools. Wnt (talk) 06:31, 27 June 2011 (UTC)[reply]
As a past participant in the F-plan diet, I can tell you that juicing oranges removes the fibre from them and leaves you with the water and other bits. If you want to get the benefit of the orange you need to eat the whole flesh (OK you can leave as much as the white pith as you wish to) and not to juice it. So real fruit juice could have the same effect, particularly if you're not drinking a great deal anyway.--TammyMoet (talk) 12:36, 27 June 2011 (UTC)[reply]
Interesting. When I get real orange, grape, or apple juice, it's not uncommon for me to run through 1-2 liters in 24 hours, but it never caused this effect. But I'm getting the impression the underlying biology is more variable than I thought. Frankly, I'd been thinking that the reaction might be universal and widely known, since after all there are things like olestra (and I think adulterated olive oil...) which are widely sold and which have equally disturbing effects. Wnt (talk) 15:04, 27 June 2011 (UTC)[reply]
Fruit juice containing pulp contains dietary fibre. ~AH1 (discuss!) 01:31, 30 June 2011 (UTC)[reply]

WIMPs

There is a huge amount of conjecture and speculation about weakly interacting massive particles in the literature, going back to mirror matter hypotheses in the 1950s. Has there ever been any evidence for their actual existence? 76.254.22.47 (talk) 07:13, 27 June 2011 (UTC)[reply]

Have you read Weakly interacting massive particles#Experimental detection? Red Act (talk) 07:21, 27 June 2011 (UTC)[reply]
Neutrinos are WIMPs so the answer is yes! But so far no smoking gun evidence for other kinds of WIMPs has been found. Dauto (talk) 07:54, 27 June 2011 (UTC)[reply]
That's not true and the Wiki-articles on this subject are out of date. The DAMA-Libra results which were originally received by a lot of scepticism have been confirmed by the COGENT results, and very recently COGENT has also reported an annual modulation. These are consistent with light WIMPS of a mass of about 8 GeV to which other DM direct detection experiments are not sensitive to. There are a lot more physicists taking these results serious than a few years ago. Because you first had the DAMA results, then the confirmation of these results by DAMA-Libra and then later the completely independent experiment COGENT. Count Iblis (talk) 15:08, 27 June 2011 (UTC)[reply]
Thanks for that information. I had heard of the DAMA results but wasn't too impressed. I will definably look into the COGENT experiment which I hadn't heard of before. Dauto (talk) 16:28, 27 June 2011 (UTC)[reply]
After reading about CoGeNT (Which seems to be part of the CDMS II experiment), I have to say that I don't think that is the smoking gun evidence just quite yet, though we may be getting tantalizingly close to it. Dauto (talk) 19:01, 27 June 2011 (UTC)[reply]
Just read an article in Science about WIMPs yesterday. It's in their news section, so it should be pretty accessible to the lay reader. Bhattacharjee, Y. (2011). "Possible Sighting of Dark Matter Fires Up Search and Tempers". Science. 332 (6034): 1144–1147. doi:10.1126/science.332.6034.1144. PMID 21636756. NW (Talk) 15:46, 29 June 2011 (UTC)[reply]

Difference between curd and yoghurt

difference between curd and yoghurt... their nature and formation..... acidic nature.... — Preceding unsigned comment added by Simplyds (talkcontribs) 12:13, 27 June 2011 (UTC)[reply]

Have you read the Wikipedia articles on curd and yoghurt? The differences are easy to discover. Oddly, the acidity of these foodstuffs seems harder to determine. After a bit of a look round I found this which is a bit heavy but has info on yoghurt, and this one has info on milk curd. Richard Avery (talk) 12:55, 27 June 2011 (UTC)[reply]

Capturing CO2

Could the CO2 expelled by a car be captured? Wikiweek (talk) 12:18, 27 June 2011 (UTC)[reply]

Sure, by attaching a hose to the exhaust pipe and routing it somewhere. ←Baseball Bugs What's up, Doc? carrots12:57, 27 June 2011 (UTC)[reply]
Yes Buggs, I'm thinkin' the "somewhere" leaves your response a bit open-ended, and not even clever. Caesar's Daddy (talk) 13:13, 27 June 2011 (UTC)[reply]
I've seen this done in garages, when the mechanics have to run the engine for awhile to test something. The hoses presumably route the CO, CO2, soot, and whatever else, to the outside of the building. Given that, it ought to be possible to capture it. Although, as noted, it's not "pure" CO2 emitting from a car, but a variety of things. So it might be possible to capture the contents of the exhaust, but by what process would you separate the various products from each other? ←Baseball Bugs What's up, Doc? carrots13:37, 27 June 2011 (UTC)[reply]
Somewhere to a CO2 parallel universe, where it disappears? But would it be practical? You'll still need some way of compressing it... — Preceding unsigned comment added by Wikiweek (talkcontribs) 13:14, 27 June 2011 (UTC)[reply]
You can bubble the CO2 through lime water which sequesters it as Calcium carbonate. This is a simple kind of carbon dioxide scrubber which I will let people read and follow links from on their own time. Attaching such systems to cars, however, is impractical and expensive, which is why it isn't done. --Jayron32 18:30, 27 June 2011 (UTC)[reply]
Several by-products of combustion are already captured in a catalytic converter, but CO2 is not one of them because out of all the by-products, CO2 is one of the least harmful. Vespine (talk) 00:42, 28 June 2011 (UTC)[reply]
Catalytic converters work by reducing nitrogen and sulfur oxides to less harmful gasses. Such oxides make up a miniscule amount of the exhaust, which is still mostly carbon dioxide; so to deal with the CO2 would require a MUCH larger system. Furthermore, the primary danger of CO2 is as a greenhouse gas, and its not clear that a reduced product (in this case likely methane, CH4) would be any less harmful. The problem with carbon sequestration in calcium carbonate is that you have to, very frequently, change out the chalk and replace the lime. Quite messy business for something you want to just "work" without any outside help, like a catalytic converter does. --Jayron32 02:55, 28 June 2011 (UTC)[reply]
Note that a catalytic converter does not 'capture' anything. As the name implies, it contains one or more large-surface-area catalysts that encourage the conversion of certain exhaust components into less-noxious chemicals. (This generally includes the combustion of unburned or partially-burned fuel, the oxidation of carbon monoxide into less-toxic carbon dioxide, and the breakdown of nitrogen oxides into nitrogen and oxygen.) TenOfAllTrades(talk) 14:56, 28 June 2011 (UTC)[reply]
It's not very practical to capture the carbon dioxide from the tailpipe as the car is being driven down the street, for the reasons listed above. A more practical approach may be to use carbon credits and carbon trading. Under such a system, anyone driving (or perhaps manufacturing or selling) a car or, better yet, refining or selling gasoline for the car, would have to pay a company to extract the equivalent amount of carbon dioxide from the air and sequester it. This approach has the possibility of being far more efficient, but requires the political will to pass the legislation to make it mandatory. I believe there are existing companies which will currently do this on a voluntary basis, but, of course, most drivers won't participate. StuRat (talk) 17:29, 28 June 2011 (UTC)[reply]
Lithium hydroxide is one of the few substances that can capture CO2 at the source. ~AH1 (discuss!) 01:29, 30 June 2011 (UTC)[reply]
But it still wouldn't be very practical to haul a tank of it around with you, with the need to replace it at each fill-up. StuRat (talk) 17:07, 30 June 2011 (UTC)[reply]

What are the main reasons it will fail? 20.137.18.50 (talk) 12:33, 27 June 2011 (UTC)[reply]

Well, it might fail to solve some of the technological challenges required to make fusion happen in the first place. Or it might achieve a repeatable fusion reaction that consumes more energy than it releases. Or it might solve all those problems, but still not be an economically feasible process. Gandalf61 (talk) 12:49, 27 June 2011 (UTC)[reply]
Private companies based on fusion have failed in the past because the capital R&D costs are very high and the guarantee of economical success is nonexistent. They have an additional downside from government or university work in that they are reliant on investment funds but they are trying to develop proprietary technology. So generally they need hype but have difficulty providing the details about how they made it happen, which is a sure recipe for suspicion. There was an ICF company in the early 1970s that failed after cycles of this sort — it turned out that ICF was hard, and that generating sustained research funds from private donors was hard. Universities and governments have typically dominated in this field because their funding mechanisms are not profit-based. You can still get useful results even if you don't get closer to getting a profit in those contexts, but that is not a good business model.
This is a separate question from the specific technical one. The general difficulty with fusion is that the tolerances for imperfections (either in keeping the plasma off of the walls in MCF, or compressing it in ICF) are very low, for any imperfection rapidly leads to either incredible inefficiency or rapid cooling, both of which make generating a gain in net energy unlikely. Now I don't know much about Magnetized target fusion but it seems to be an attempt to average out the difficulties of both. Whether it actually works to that end, or whether it simply means you are trying to solve two different and difficult problems at the same time, remains to be seen, I think. The track record for fusion is not great: every problem solved usually results in the discovery of two more problems. This is why the two current most favored efforts (NIF and ITER) are basically based on the idea of scaling it up to monstrous sizes so that hopefully the imperfections become less important. --Mr.98 (talk) 13:52, 27 June 2011 (UTC)[reply]
I don't like your will there. That's very pessimisitic. It could fail, for a variety of reasons from physics to politics, but that's far from certain. i kan reed (talk) 17:05, 28 June 2011 (UTC)[reply]
I think the OP is just expressing the fact that so far every attempt at commercializing fusion (much less getting even net energy) has failed over the past five decades. There are little reasons to be optimistic with fusion — the history of fusion research is filled with people saying "this shouldn't be too hard — give us 5 years!" again and again and again and again. It turns out that controlled fusion is genuinely hard. --Mr.98 (talk) 18:44, 28 June 2011 (UTC)[reply]
What's so hard about putting a star in a bottle ? :-) StuRat (talk) 19:13, 28 June 2011 (UTC) [reply]
Five years? Everyone knows commercial fusion power is twenty years away -- and has been for the past half-century. --Carnildo (talk) 00:40, 29 June 2011 (UTC)[reply]
University/lab scientists say 20 years. People wanting to commercialize it say 5 years. Which is a nice reflection of the different funding forces playing on both of them — one says "fund more basic research and you'll get this cool technology," the other says, "invest today, earn money soon!" --Mr.98 (talk) 00:44, 29 June 2011 (UTC)[reply]

bond angles

how "flexible" are bond angles (or equilibrium bond angles, because they can move)? I mean in large molecules like polymers, they can easily bend (such as polyethylene) and we see a lot of images showing structures like DNA bending as if its a simple rope or something.also, the covalent bonds in liquid glasses such as SiO sre still present but apear to be somehow looser than the solid form.. if the bond angles (or equilibrium bond angles) are more flexible when the olecules are larger,why is it that way. is everything about molecular geometry only true for smaller molecules? thanks.--Irrational number (talk) 12:44, 27 June 2011 (UTC)[reply]

  • Some bonds are free to move as a swivel. allowing large molecules to bend.
  • Large molecules have many bonds. The slight flexibility of each of them can add up to a large flexibility for the larger molecule.
Dauto (talk) 17:14, 27 June 2011 (UTC)[reply]

Do these materials(large molecules) naturaly tend to bend too?-Irrational number (talk) 18:26, 27 June 2011 (UTC)[reply]

There are two conflicting things here. First, a long simple alkane is indeed very floppy, and bends quite alot. However, that bending only works if the molecule is dimensionally constrained; essentially long straight chains are the "floppiest". When there is a network of bonding within a molecule in three dimensions, it can constrain the bonding. The carbons in an alkane are in the same hybridization as the carbons in diamond, and yet diamond is very much NOT flexible. In DNA, the three-dimensional structure constrains the DNA chain into a relatively rigid double helix, though the entire chain does have some flex to it. Look at other macromolecules like proteins. Some proteins are very flexible, while others are not. The difference is the presence of large numbers of cysteine bases, which form disulfide bonds and constrain the structure of the protein; proteins with lots of disulfide crosslinking (like keratin) tend to be relatively inflexible. --Jayron32 18:28, 27 June 2011 (UTC)[reply]
There are cubane, propellanes, cyclobutane at 90 degrees ... even cyclopropane at 60 degrees. It's hard to think of a molecule that forces C-C bonds at a smaller angle than that, but if you do ... odds are someone's done research on it. ;) The bonds have a preferred angle, yes, but generally carbons seem to prefer association of any sort over none at all. But the strain on such structures is enormous - of course, in a long piece of plastic, the deformation of any one bond is absolutely miniscule by comparison. Wnt (talk) 18:46, 27 June 2011 (UTC)[reply]
Cyclopropene beats 60° :). DMacks (talk) 12:51, 29 June 2011 (UTC)[reply]
Cyclopropyne doesn't appear to be an energy-minimum, but if you're willing to look more broadly (simple octet/4-bond main-group atoms other than carbon), silacyclopropyne does exist and the C–Si–C angle is around 41°.(doi:10.1021/ja960762n) DMacks (talk) 13:50, 30 June 2011 (UTC)[reply]

how did biblical figures probably walk on water

if not by magic, then what is the most likely mechanism biblical figures used to walk on water? — Preceding unsigned comment added by 188.29.96.144 (talk) 13:38, 27 June 2011 (UTC)[reply]

There is not universal agreement that Jesus was literally walkig on the water. ←Baseball Bugs What's up, Doc? carrots13:40, 27 June 2011 (UTC)[reply]
They probably didn't... AndyTheGrump (talk) 13:42, 27 June 2011 (UTC)[reply]
The could have used floating sandals, as in the illustration in our article on Walking on water.--Shantavira|feed me 13:49, 27 June 2011 (UTC)[reply]
The article Jesus' walk on water gets into specifics of the Bible story. ←Baseball Bugs What's up, Doc? carrots13:52, 27 June 2011 (UTC)[reply]
I think Jesus walked on water using the same levitation mechanism by which Yoda levitated, i.e., the levitation was enabled by the imagination of the storyteller. But see levitation for real levitation mechanisms, and levitation (paranormal) for other mythological levitation. Red Act (talk) 14:08, 27 June 2011 (UTC)[reply]
A sandbar close to the surface of the water could give the appearance of walking on water especially if seen from another water-going vessel—perhaps at a distance, and perhaps with the lower portion of the body blocked from view by the water-going vessel upon which one finds oneself. Bus stop (talk) 19:37, 27 June 2011 (UTC)[reply]
Maybe they thought life is a state of mind. Richard Avery (talk) 19:48, 27 June 2011 (UTC)[reply]
Modern magicians use a clear plastic support, just below the water. Plastic wasn't available then, so perhaps glass was used. Unlike the sandbar, this can fool people right there, as clear objects can become virtually invisible, once submerged, especially with waves at the top. Also note that in Biblical times it was common for religious figures to use magic tricks to convince people they had God(s) on their side. The magic contest between Moses and the Egyptian high priest, where each turned their staff into snake(s), shows this nicely (some Christian fundamentalists might argue that Moses' trick was the real thing, but who argues that the Egyptian priest was really using the power of Egyptian gods ?). StuRat (talk) 19:54, 27 June 2011 (UTC)[reply]
You have to be careful with that trick. The nature of waves in shallow water is different from deeper water. In Superman II there was a scene that showed the evil General Zod walking on water. They got a little too close with the camera, and although you couldn't see the platform he was walking on, you could see its effect in the rippling of the water. ←Baseball Bugs What's up, Doc? carrots23:39, 27 June 2011 (UTC)[reply]
But, of course, you were merely seeing the effect on the waves from the supporting force field. StuRat (talk) 04:21, 28 June 2011 (UTC) [reply]
Egyptian fundamentalists of course. If Wikipedia had been around in Old Testament times, there might have been a heated debate. Wanderer57 (talk) 21:28, 27 June 2011 (UTC)[reply]
If the internet had been around in O.T. times, God would simply have posted the Ten Commandments in His blog. ←Baseball Bugs What's up, Doc? carrots23:35, 27 June 2011 (UTC)[reply]
If it was a wiki, I'd hate to think of all the edit wars on that one. StuRat (talk) 04:21, 28 June 2011 (UTC) [reply]
He would probably have added "Thou shalt not annoy others with your constant twittering." Googlemeister (talk) 13:21, 28 June 2011 (UTC) [reply]
And his Admin policy would have been strict. "Disemvowellement? No, I think you misheard." {The poster formerly known as 87.81.230.195} 90.201.110.117 (talk) 17:18, 28 June 2011 (UTC)[reply]
There are two ways that humans today can walk on water: either by walking on ice, or by running very quickly on floating substances such as lily pads or thin wood. ~AH1 (discuss!) 01:27, 30 June 2011 (UTC)[reply]
In Jesus the Man, Barbara Thiering presents an interesting idea about the origin of "walking on water". According to her, it was routine for priests to perform a certain ceremony on a boat that involved people wading from the shore to the boat. By contrast, the priest, dressed in robes and needing to maintain dignity, could not wade through the water, and so a low-lying jetty was built solely to allow the priest to walk out to the boat. To the onlookers from the shore, it looked like the priest was walking on water, and they used that phrase to joke about priestly superiority. A certain event involving Jesus and the jetty led to the phrase being adopted for Jesus (and embellished in documented form, according to Thiering). Johnuniq (talk) 03:43, 30 June 2011 (UTC)[reply]

painkillers for waxing

What would be more effective pain relief for waxing of intimate areas? Paracodal or ibuprofen? I've been advised to take something before my appt and these are the two options in my drawer! Thanks. (just so you know, I take both occasionally and am not allergic to either and have never had any adverse reactions in the past and this is not a request for "medical advice", you can just give me science!) 195.27.52.146 (talk) 15:15, 27 June 2011 (UTC)[reply]

It really is a request for medical advice, actually. Who advised you? In many countries only a medical practitioner would be able to advise. In others, perhaps a pharmacist. Itsmejudith (talk) 15:51, 27 June 2011 (UTC)[reply]
I feel that this is a specific question about the action of the two drugs. In this case, there is a very clear answer. Ibuprofen primarily reduces inflammation and fever. Neither of those will be of much help in reducing upcoming pain in sensitive nerve endings. Paracodal contains paracetamol, which specifically reduces pain by reducing the sensitivity of nerve endings. Both are "pain relievers", but they relieve different kinds of pain. If this doesn't completely answer the question, please delete it and mark this as a true request for medical advice. -- kainaw 18:41, 27 June 2011 (UTC)[reply]
This sounds like a request for medical advice to me. I have no idea, for example, whether the practitioner might be relying on your yelps and screams as feedback about whether he/she is pulling too hard, and by suppressing them you could make something gruesome happen. IMHO if something is so painful you need a drug to suppress the pain, your body is trying to gently hint to you that perhaps it is not a good idea. Wnt (talk) 18:50, 27 June 2011 (UTC)[reply]
I don't think you need a medical license to suggest cosmetics, so I would go with a skin cream or shaving cream containing menthol and/or eucalyptus and/or aloe. The menthol a pain-killer, the eucalyptus is an antiseptic to prevent infection, and the aloe is a painkiller and will help you heal after. The advantage of a topical cream is that it's much stronger where you need it, while anything you take orally is distributed around your body, so very little of it will actually go to the skin in question. I do agree with Wnt, though, that if they advise stronger painkillers, they aren't doing it right. It shouldn't be that painful. StuRat (talk) 20:08, 27 June 2011 (UTC)[reply]
Why not use Lidocaine/prilocaine?

The lidocaine/prilocaine combination is indicated for dermal anaesthesia. Specifically it is applied to prevent pain associated with intravenous catheter insertion, blood sampling, superficial surgical procedures; and topical anaesthesia of leg ulcers for cleansing or debridement.[3] Also, it can be used to numb the skin before tattooing as well as laser hair removal.

Count Iblis (talk) 22:32, 27 June 2011 (UTC)[reply]
Kainaw is right, that's just what I wanted to know. To the rest, waxing is notoriously painful and taking a couple of painkillers before doing it is pretty much standard advice from the practitioners. Wnt—there's no such thing as "pulling too hard" when it comes to waxing! That's the whole point.  :) Thanks for all the responses! 195.27.52.146 (talk) 06:44, 28 June 2011 (UTC)[reply]

communication over vast distances

Is it in any way possibly to send information over vast distances (in a reasonable amount of time). Even better, is this also applicable to traveling? 66.229.227.191 (talk) 15:19, 27 June 2011 (UTC)[reply]

The question is vague. By typing this comment, I'm sending information to people in distant countries, if they happen to be looking at this page. By using my cell phone, I can talk to those people. Did you mean vaster or faster than that? Michael Hardy (talk) 20:43, 27 June 2011 (UTC)[reply]
No form of communication or transportation is faster than the speed of light, which severely limits interstellar travel. Red Act (talk) 15:39, 27 June 2011 (UTC)[reply]
A wormhole would be useful but there is no reason to believe that they actually exist. Dauto (talk) 17:19, 27 June 2011 (UTC)[reply]
The speed of light is fast enough if you transmit yourself using the message. Ultimately, we are just information stored in the brain and all that can in principle be transmitted. If the message that defines you travels 10,000 light years and upon arrival you are recreated using the received message, you would not experience the 10,000 years. Count Iblis (talk) 17:32, 27 June 2011 (UTC)[reply]
You wouldn't experience anything. If such technology existed, allowing a copy of a human brain's "state" to exist, it would be reasonable to describe it as a fork(). A copy of the state would be created and begin new processing. But it does not change the processing-state of the original copy. Needless to say, hypothetical technology to serialize the complete state of a human brain, transmit it by any method, and restore functionality by running it on a "virtual machine" is still science-fiction. Much work remains to precisely define the type of "machine" and its "instruction set" or "state." Even once these tasks are complete, building a replica and designing a "scanner/copier" will be additional, complex tasks. Nimur (talk) 17:43, 27 June 2011 (UTC)[reply]
Add to that the fact that you would only be able to travel to places that have a receiver. Dauto (talk) 19:28, 27 June 2011 (UTC)[reply]
If it takes 10,000 years to get there, they might have a receiver built by then. ←Baseball Bugs What's up, Doc? carrots23:33, 27 June 2011 (UTC)[reply]
Indeed, so if e.g. the Earth were to face some disaster, we could transmit signals that contain the information to recreate our civilization elsewhere. That would be easy to do if we have transformed to a machine civilization. If the signal is powerful enought it can be picked up millions of lightyears away in some nearby galaxy. They can then download our programs and eventually our entire civilization can be rebuilt there.
A limiting factor here is the data rate. Perhaps the best strategy is to have a radio beacon repeatedly transmitting an easy to decode message that tells listeners to look out for a laser beam. Using a visible light laser, one can transmit more than 10^14 bits/second. So, more than 10^21 bits of information can be transmitted per year and if we use higher frequency beams much more than that.
The power of radio transmitter and the laser beam must be large, but that's not a problem for a machine civilization capable of using all the power that the Sun emits. The laser beam can be split into separate parts that are aimed at many of the nearby galaxies. The beam divergence can be made small enough such that the beam's width is exactly the size of the galaxy upon arrival (the brighness of the beam would then be similar to the apparent brighness of a star at a distance of the order of the diameter of that galaxy, so it would be easy to detect).
Our survival then depends on there being just one civilization capable and willing to recreate us among the many hundreds of nearby galaxies. Count Iblis (talk) 00:16, 28 June 2011 (UTC)[reply]
If Earth received instructions on how to fabricate individuals of some dying alien species, along with DNA sequences (or the alien equivalent) of numerous exemplars of the species, and especially if they were cute and nonthreatening (dolphins, seaturtles, pandas, Love birds), some humans would feel sorry for them, or want to adopt and raise them, or would hope to gain financially or militarily from their special abilities, or to publish articles about them in scientific journals, or exhibit them in zoos, or whatever. It is quite likely that someone would build the machine and feed in the DNA sequences, producing some living exemplars of the species. Of course their civilization might really be be aggressive and flourishing rather than dying, and the cadre of synthesized individuals might be genetically coded such that when they are grown they exterminate, enslave or replace the native population or open a port of some sort to allow a wholesale invasion. Such unintended consequences are a likelihood if a civilization does what someone in a distant star system tells them to do. On Earth, people have carried out malicious instruction from random phone callers such as putting Vaseline and alcohol on their hair and setting it on fire, cutting off their hair, [2] or cutting off someone else's hair, soaking the phone in the toilet, or strip searching a teenage employee at a fast food restaurant. The "call from space" might also be malicious or a prank. Even if the authorities suspected it was not on the level, that the senders might be con artists, pranksters or hostile militarists, they would be likely to build it for fear that otherwise the "other side" or terrorists will build it and gain some competitive or military advantage from the new technology. Those who send out technological information to other stars might be idealistic and altruistic benefactors, or they might have ulterior motives, such as a xenophobic wish to exterminate all other intelligent lifeforms. An "interstellar hoax message" comparable to the ones which got the women to cut or ignite their hair might tell us that there is some "peril from space" (alien invasion or bacteriological weapon coming from space) and that we need to launch a crash program to protect ourselves, or that we need to kill all marine mammals, or modify the DNA of the next generation of humans, or vaccinate ourselves with some bogus vaccine they helpfully sent instructions for. The basic interstellar hoax message would tell us to build some gadget which will make us happier, healthier, wealthier, or more powerful, complete with demo video showing the "cornocopia" will feed everyone, or the "filter" will unpollute the air and water, or the machine will eliminate infectious diseases, or the "antigravity device" will make it easy to get around, or the "energy machine" will eliminate powerplants. Industrialists and governments could convince themselves that it would be "Ok to build a small demonstration model" and that they could "turn it off at the first sign of trouble." Hilarity ensues: "IT'S NOT STOPPING! IT'S GETTING BIGGER! AIEEE!" Many classic con games could be modified to be motivating. Most Earthly hoax messages seem to just be intended less to provide actual gain to the sender than to make people look silly, run around and waste resources, like fake distress calls from nonexistent sinking vessels, or fake bomb threats. Edison (talk) 16:25, 29 June 2011 (UTC)[reply]


If light-speed communication is too slow for you then superluminal communication is the page to look at for more information. Apparently the prospects are not good. --Antiquary (talk) 19:21, 27 June 2011 (UTC)[reply]

Biology

Why do some birds migrate even though they are warm blooded ? — Preceding unsigned comment added by Ssmagic (talkcontribs) 16:27, 27 June 2011 (UTC)[reply]

It doesn't have anything to do whether an animal is warm blooded or cold blooded. Many species of all kinds migrate. See animal migration for details. --- Medical geneticist (talk) 16:32, 27 June 2011 (UTC)[reply]
Humans are warm-blooded, but we still find it desirable to take many steps in order to avoid being cold. Why should it be any different for animals? There are many different adaptations to cold, including migration and hibernation. Dragons flight (talk) 17:01, 27 June 2011 (UTC)[reply]
Humans also migrate, at least in their older years... --Jayron32 18:21, 27 June 2011 (UTC)[reply]
Apart from seeking more comfortable temperatures, some bird species have to migrate because the availability of their preferred or obligatory food sources are greatly affected by the changing seasons. If, for example, a bird lives on insects, it would likely find very slim pickings during a Northern European winter, making migration to, say, North Africa, advantageous. Our article Bird migration describes in detail this and several other reasons for the phenomenon. {The poster formerly known as 87.81.230.195} 90.197.66.204 (talk) 19:46, 27 June 2011 (UTC)[reply]
Another frequent reason for migration is reproduction. They often migrate from a wide area to a small one, where their greater population density helps them find a desirable mate. Then, there is sometimes "safety in numbers" where they raise the chicks, or perhaps they breed in an inaccessible area to deter predators. StuRat (talk) 20:29, 27 June 2011 (UTC)[reply]
In general, birds migrate because of the reasons above, not because it's cold. There are plenty of birds that hang around in winter when there's a food source. You might say the cold and snow are indirect reasons for the migration. ←Baseball Bugs What's up, Doc? carrots23:31, 27 June 2011 (UTC)[reply]
I suppose you could argue that those species which migrate to warmer climates in winter never developed (or lost) the adaptations that would allow them to survive cold weather (right up to penguins that can survive winter in Antarctica). So, at this point, they do need to migrate to avoid the cold, among the other reasons. StuRat (talk) 04:17, 28 June 2011 (UTC)[reply]
Penguins migrate to a degree too though, don't they? Googlemeister (talk) 18:35, 28 June 2011 (UTC)[reply]
Yes, but for other reasons, not to stay warm. StuRat (talk) 22:15, 28 June 2011 (UTC)[reply]
"The swallow may fly south for the winter, yet it is no stranger to our land..." Albval (talk) 19:36, 28 June 2011 (UTC)[reply]
Be amazed by the journeys of the arctic tern, typically about 2 million km in a lifetime, commuting from the arctic to the antarctic. Mikenorton (talk) 19:45, 28 June 2011 (UTC)[reply]

hello (identifying #7 plastic)

how do they tell #7 plastics apart at recycling centers — Preceding unsigned comment added by Tsp12345 (talkcontribs) 19:38, 27 June 2011 (UTC)[reply]

That's a good question. The plastic recycling article is rather hazy on the subject, simply saying that they're separated based on their resin identification code. This article says the manual sorting process is too expensive and isn't sufficiently reliable, and talks about two technologies that can identify some plastic types. It's not clear if these automatic methods are in widespread use. In my own area, it seems they only want drinks bottles; everything else (tubs, trays, lids, caps, films, bags, etc.) they don't take. I imagine that if (or when) most retail products contain RFID tags, the separation of some post-consumer waste (bottles, tubs, trays at least) may become more easily automated. -- Finlay McWalterTalk 19:51, 27 June 2011 (UTC)[reply]
Some recycling programs will only accept properly marked (i.e. with the correct code) plastics for recycling. Some plastics, if kept with the same type, can be melted down and reused in the same application; however this requires proper sorting. While "pure" plastics can fetch greater prices on resale, they are also much more labor intensive to ensure proper purity; some cheaper reuses (for example, shredded and used as mulch) fetch much less on the open market, but then again are much cheaper to process. Plastic recycling has more information, and the economics of recycling are quite complex, especially when coupled with the "social pressures" to recycle even when it is not financially feasible to do so. --Jayron32 20:22, 27 June 2011 (UTC)[reply]
I added to the title to make it actually useful. StuRat (talk) 20:25, 27 June 2011 (UTC)[reply]
I found a few sources via Google Books. Several of them say that Near Infrared (NIR) and Middle Infrared (MIR) spectroscopy can be used with some success, but that it doesn't work with plastics that have black/grey dyes in them, because those absorb the NIR wavelength; also MIR cannot be used for high-speed processing.[3][4][5][6] Other techiniques measure the density or melting point, or use chemical markers.[7][8]] ... Anyways, I would suggest searching Google books for identifying plastics recycling, and you'll probably turn up quite a bit more information. Cheers. ~ Mesoderm (talk) 20:36, 27 June 2011 (UTC)[reply]
Some plastics labelled as recyclable are indeed not. Those with very low rates of recylcling include #3, #6 and #7. ~AH1 (discuss!) 01:24, 30 June 2011 (UTC)[reply]

weird plastic taste in coffee

Hi, I'm a total coffee philistine, but one thing I can't handle is the plastic taste that you get from some containers. Basically I brew up several cups, and store them in my plastic container (Sistema brand, in Australia), but of late, I've noticed the ghastly plastic gunky taste is getting in the coffee, even though it didn't originally. I've been using them for several months without incident, and now, suddenly, nearly every coffee comes out awful. Why does this happen? Surely plastic is inert, so it shouldn't get into food. Has this problem ever been investigated with plastic containers? Also, is there any cure? It's been emotional (talk) 21:04, 27 June 2011 (UTC)[reply]

I used to drink coffee out of various plastic and foam containers and noticed the same thing. Given that many plastics (or byproducts of burning/heating them) act as endocrine disruptors or mutagens, I didn't feel like taking any chances and started drinking out of ceramic, glass, or metallic containers. I no longer have to deal with the unpleasant plastic taste (which I notice every time I drink out of plastic containers now), and I don't have to worry about the health effects of drinking a hot liquid out of a plastic container. ~ Mesoderm (talk) 21:13, 27 June 2011 (UTC)[reply]
I agree with glass (not leaded crystal) or ceramic (unpainted ceramic, that is). Metal, however, can also react with beverages, so they often coat this inside of metal containers with plastic to prevent this, and we're back to the original problem (unless you have a gold or platinum bottle :-) ). I use glass, myself. Specifically, I buy Everfresh juices and reuse the glass bottles: [9]. StuRat (talk) 22:32, 27 June 2011 (UTC)[reply]
To answer your question, there are several chemicals that can leach out of plastic bottles into the contents. More leaching is expected with more heat, more flexing of the bottle, more time, and more acidic content (a highly alkaline drink might be a problem, too, but there aren't many of those). In your case, pouring hot coffee into it might be the problem. StuRat (talk) 22:35, 27 June 2011 (UTC)[reply]

Thanks, very astute of you, StuRat. Thanks to all of you. It's been emotional (talk) 23:11, 27 June 2011 (UTC)[reply]

Don't they have ceramic coffee pots coffee pots in Australia? Or even teapots? (I see that "coffee pot" re-directs to "Coffeemaker", which are of course not coffee pots. Coffee pots are similar to tea pots, but designed for coffee. Usually taller and more cylindrical than teapots in my experience). 2.97.219.42 (talk) 10:15, 28 June 2011 (UTC)[reply]
In US English, at least, a coffee pot (that's 2 words in US English) is the container into which the freshly brewed coffee pours from the coffee maker (also 2 words), and a tea pot (also 2 words) is where the tea is steeped. So, neither is the storage container asked about here. StuRat (talk) 13:14, 28 June 2011 (UTC)[reply]
The US is not the whole wide world, and people often do things differently in different countries. 92.29.120.26 (talk) 20:28, 28 June 2011 (UTC)[reply]
And I never said that it was, did I ? If there's a country where a "coffee pot" (as 1 word or 2) or a "tea pot" (as 1 word or 2) are used for storage of beverages, then I'd like to know that. StuRat (talk) 20:42, 28 June 2011 (UTC)[reply]
Well, Britain, Europe, India, China, and many more. Cofffee pots are particularly used in Arabia I understand. Its called an "X"-pot because its got "X" in it. Perhaps you are confusing them with kettles, which are only used for heating water. 92.29.120.26 (talk) 21:04, 28 June 2011 (UTC)[reply]
No, I'm not confusing those. See below. StuRat (talk) 22:13, 28 June 2011 (UTC)[reply]
Typical UK coffee pots - here  Ronhjones  (Talk) 21:28, 28 June 2011 (UTC)[reply]
That's what I'm thinking of. Those are for serving coffee, not for storing it. My reading of the Q is that they want to store the coffee, and possibly transport it to another location, and therefore need a container which can be sealed for that purpose, such as a thermos. StuRat (talk) 22:10, 28 June 2011 (UTC)[reply]
Perhaps they serve coffee in a coffee pot in the US, but on this side of the Atlantic it is considered uncouth to put the spout of the coffee pot in your mouth: over here the coffee is always served in a cup or mug. Something to remember if you ever visit our sceptered isle. 92.24.141.227 (talk) 21:08, 1 July 2011 (UTC)[reply]
Does "serve" have a different meaning there, too ? Here a coffee service or tea service includes everything placed on the table, such as the pot, cups, and saucers. StuRat (talk) 03:04, 2 July 2011 (UTC)[reply]
You can do whatever you like in the US, but over here such behaviour is frowned upon. 92.29.127.122 (talk) 13:04, 2 July 2011 (UTC)[reply]

is it any easier to catch a bullet at the top of its arc if you throw it up than if you shoot it up?

If I take a bullet in my hand, and throw it up a couple of feet for you to catch, is it any easier for you to catch it than if we repeat the same thing, but, in a very still place, you happen to be on top of a cliff and the arc of the bullet I shoot from somewhere far far away looks, relative to you, exactly the same on paper as when I threw it up to you from a couple of feet below? I mean, because the top of the parabola should look the same. — Preceding unsigned comment added by 188.28.68.234 (talk) 22:10, 27 June 2011 (UTC)[reply]

From a pure physics standpoint, velocity is velocity; the round has no memory that it used to be going very fast; and so, near the top of a parabola, vertical velocity approaches zero. "Engineering details" may confound this simplistic viewpoint: a round fired from a gun will be hot; it may be wobbling; it may have deformed during firing or flight. It will also be hard to aim very precisely due to error propagation (a longer time in flight means that tiny error in aim will result in large error in position). The safety factor should be considered, too - if you miscalculate, or if wind, non-ideal turbulence, or any other parameter changes the expected peak location of the trajectory, your experimenter will be in a very dangerous position. Nimur (talk) 22:18, 27 June 2011 (UTC)[reply]
Well, the bullet still wouldn't be going very fast, even if your calcs are off a bit. However, the variability is enough that you couldn't reliably get the bullet to reach the peak of it's trajectory within reach of the target person on top of the cliff. So, it would be easier to catch if hand thrown. StuRat (talk) 22:25, 27 June 2011 (UTC)[reply]
Also if the bullet is shot out of a rifle it might be spinning pretty fast, which might make it harder to grab. Rckrone (talk) 23:15, 27 June 2011 (UTC)[reply]
I think what the OP meant by "easier" is purely the act of nabbing the bullet, given that you just happen to be in the right place at the right time. At the very top of its arc, at least for one moment, it will have zero or near-zero vertical velocity, before it starts to fall again. So if you were next to the bullet and had a butterfly net or something, grabbing it should be just as "easy" as if it were thrown up in the air. Obviously, this only works if you've thrown or shot it pretty much straight-up. ←Baseball Bugs What's up, Doc? carrots23:26, 27 June 2011 (UTC)[reply]
The difference that comes to my mind is that you couldn't see it if it originated at a distant point. In one instance you could maintain eye contact for the length of the trajectory. In the other instance you would only be able to establish eye contact at a fairly late point in its trajectory. This would introduce an added difficulty. Human vision is inadequate to the task of tracking a fast-moving small object especially at a considerable distance. While the object would still be small if tossed from only a few feet away, the tracking of such an object by the eyes of humans is still within normal operating range. Bus stop (talk) 23:33, 27 June 2011 (UTC)[reply]
You might, if it were painted international orange or something. Maybe the OP needs to clarify what he means by "easier". My concept of this would be that you could fly like Superman and be able to spot the bullet and to be at the top of the arc and grab it. However, Superman could grab it as soon as it leaves the rifle without doing himself any harm (unless it was made of kryptonite). So the OP needs to comment further. ←Baseball Bugs What's up, Doc? carrots23:49, 27 June 2011 (UTC)[reply]
Let's formulate this as a problem. Let's say we have a .38 Special, with a muzzle velocity of about 600 feet per second, which is pointed upward and fired. How accurately must the muzzle velocity and direction be controlled in order to produce an error of less than 1 foot in any direction in the location of the trajectory peak? Extra credit: how does the necessary directional accuracy compare to the maximum possible accuracy of a .38 Special? Looie496 (talk) 23:49, 27 June 2011 (UTC)[reply]
No one has specifically said it so I'll add it. Both bullets will be experiencing a constant downward acceleration of 9.8m/s2 . So yes, the "top" of the parabola of the fired bullet will be identical to the parabola formed by a thrown bullet from the point where the thrown bullet's velocity matches that of the fired bullet. As stated above however, this is only given "perfect" conditions of no friction or turbulence (etc...) which of course don't exist in real life. Vespine (talk) 00:30, 28 June 2011 (UTC)[reply]
A bullet shot straight up will still travel a few thousand feet. If a bullet travels 5000 feet, then to be accurate to within 1 foot (assuming there is no wind or other atmoshperic effects) you will need accuracy of .01 degrees to be within a foot. I don't think most handguns would have that level of accuracy and even a sniper rifle would be tricky. Of course a sniper rifle would probably go a lot higher then 5000 feet since it has a much higher muzzle velocity. Googlemeister (talk) 13:13, 28 June 2011 (UTC)[reply]

miami

do they have central heating in most houses in miami and what kind — Preceding unsigned comment added by Superhands99 (talkcontribs) 22:40, 27 June 2011 (UTC)[reply]

It does get chilly enough in winter to need some form of heating, but, due to the relatively low amount of heating needed, electrical heating is common (the cost of electrical heating is prohibitive in colder climates). Electric baseboard heating is one common form. StuRat (talk) 22:56, 27 June 2011 (UTC)[reply]
Miami recently set a record because 10 straight days failed to reach 65 F in January.[10] I think little heating is required. Miami has a climate of 149 heating degree days versus Detroit (6224) or Duluth (9371). Rmhermen (talk) 00:58, 28 June 2011 (UTC)[reply]
Even 65°F is a bit cold, especially for the elderly. But, of course, those are the highs. The record low is 27°F: (pick "Record Low" check box). At those temps you can die without some form of heat. StuRat (talk) 04:10, 28 June 2011 (UTC)[reply]
It's my understanding that many homes in Miami area, especially the older ones, don't have heating systems. Presumably, on really cold days (by their standards), they use space-heaters or blankets or something. ←Baseball Bugs What's up, Doc? carrots01:13, 28 June 2011 (UTC)[reply]
Or they could just use their aircos as heaters :) . Count Iblis (talk) 02:36, 28 June 2011 (UTC)[reply]
That's right. I live in Florida (Not in Miami, though) and every house I've ever lived in has has a AC/heating unit, with one single exception. Dauto (talk) 03:21, 28 June 2011 (UTC)[reply]
By AC/heating unit do you mean Air source heat pumps? Rmhermen (talk) 21:19, 28 June 2011 (UTC)[reply]
Yes that correct. Dauto (talk) 18:39, 29 June 2011 (UTC)[reply]
You could just turn a small window AC unit around and it would provide heat to the room while cooling the outside right? Googlemeister (talk) 13:26, 29 June 2011 (UTC)[reply]
Yes, that's also correct. A heat pump is nothing more than an AC. Dauto (talk) 18:39, 29 June 2011 (UTC)[reply]
But if it's too cold outside, and the A/C is operated continuously, ice would build up on it. StuRat (talk) 01:01, 1 July 2011 (UTC)[reply]

June 28

Multiple sclerosis

What is meant by t1 or t2 lesion load? how does it differ from active lesion? — Preceding unsigned comment added by 199.224.149.10 (talk) 04:32, 28 June 2011 (UTC)[reply]

I can't give a detailed answer, but I can tell you that T1 and T2 are different ways of carrying out an NMR examination. This paper describes them in detail, if you can make sense of it. Looie496 (talk) 05:15, 28 June 2011 (UTC)[reply]
To explain in better detail, see Magnetic_Resonance_Imaging#Basic_MRI_scans. T1 and T2 refer to two different MRI techniques (the MR in MRI is the same MR as in NMR and the terms (NMR and MRI) are occasionally used interchangably, though mostly NMR is reserved for the analytical chemistry technique, while MRI is used for the medical diagnostic technique. The authors of this paper are really refering to MRI). The technical details of each can be found in the article I cited, but for the purpose of the OPs question, and for the purpose of understanding that paper, the T1 and T2 lesion loads are merely the number of lesions detectable on a T1 scan versus a T2 scan. Each type of scan has different sensitivities, so they have different applications; some times you'll want to do a T1 scan and sometimes a T2 scan. The paper you cited is the results of various experiments to determine the ideal parameters for using MRI to detect multiple sclerosis lesions. --Jayron32 05:27, 28 June 2011 (UTC)[reply]

Colloquial medical expressions

When someone dies of a "hole in the heart", what did kill him? And what about the classical - "dying of old age"? Does that simply mean "old person died, don't know why"? Wikiweek (talk) 14:39, 28 June 2011 (UTC)[reply]

The former may often be ventricular septal defect -- Finlay McWalterTalk 14:43, 28 June 2011 (UTC)[reply]
Or an atrial septal defect. In a society with current medical care it would be unlikely for an adult to die of any kind of septal defect, since it would usually be repaired before significant hemodynamic consequences developed; a very large or complicated septal defect in an infant could cause death. --- Medical geneticist (talk) 22:04, 28 June 2011 (UTC)[reply]
From Death_by_natural_causes: "Old age is not a scientifically recognized cause of death; there is always a more direct cause although it may be unknown in certain cases and could be one of a number of aging-associated diseases." SemanticMantis (talk) 14:49, 28 June 2011 (UTC)[reply]
In the case of a "hole in the heart" due to injury, as in a bullet wound, it might be in any part of the heart, or perhaps nearby, say in the aorta or vena cava. In this case the victim is likely to hemorrhage to death, with the loss of blood pressure probably actually causing brain death. StuRat (talk) 16:19, 28 June 2011 (UTC)[reply]
When someone dies of a broken heart, see article broken heart syndrome. ~AH1 (discuss!) 01:19, 30 June 2011 (UTC)[reply]

Quantum superposition

Hello,

How does quantum superposition fit in with the principle of contradiction? It seems like QS can allow for a statement and its negation to be jointly true (i.e., "Schrödinger's cat is dead" and "Schrödinger's cat is alive" are both true at the same time). Does this mean the principle of contradiction is wrong?

Thank you. Leptictidium (mt) 15:09, 28 June 2011 (UTC)[reply]

The principle of contradiction is logically sound. What is not clear is what is meant when you state that the cat is alive or that it is dead. Are those two states truly logic opposites to each other? QM says that these statements are not the logic negation of each other since a third state is also possible, namely the cat might be in quantum superposition state. Dauto (talk) 15:34, 28 June 2011 (UTC)[reply]
But QS is not a state in and as of itself, it's merely a superposition of states, right? Leptictidium (mt) 15:36, 28 June 2011 (UTC)[reply]
No, it is really a real state. If |a> and |b> are two orthonormal states, then so are |a'> = 1/sqrt(2) [|a> + |b>] and |b'> = 1/sqrt(2) [|a> - |b>]. Count Iblis (talk) 15:40, 28 June 2011 (UTC)[reply]
Quantum mechanics makes a clear, definitional distinction between the state of the system, and the observable state of the system. We have an article on this concept: observable. Something that is still a superposition of states is not observable. When you measure the state, you are observing one of many possible states. Nimur (talk) 15:45, 28 June 2011 (UTC)[reply]
Sorry, but that's plain nonsense. You can transform any observable using any arbitrary unitary transform to yield another observable. That such observables (whose eigenstatstes are superpositions of the previous ones) can't in practice be measured is not an issue, in principle you can measure a cat in a superposition of being dead and alive. Count Iblis (talk) 17:40, 28 June 2011 (UTC)[reply]
Therefore, no truly logic opposites can exist, since there will always be a third state possible. Yes? --Leptictidium (mt) 15:57, 28 June 2011 (UTC)[reply]
No. Count Iblis (talk) 17:41, 28 June 2011 (UTC)[reply]


Logic opposites are possible but the opposite of an electron with spin up is NOT an electron with spin down. The logis opposite of an electron in a spin up state is an electron that is not in a spin up state but could conceivably be in a superposition of states between spin up and spin down. A familiar example might help. The logic opposite of moving northward is NOT moving southward. That opposite would be not moving northward which might mean you are moving northwestward. Dauto (talk) 16:30, 28 June 2011 (UTC)[reply]
I prefer to phrase it this way. For example, let's take electron spin in the hydrogen atom. It can either take the value +1/2 or -1/2. When it is not measured, I do not say, "the particle is both spinning with +1/2 and -1/2." Instead, I phrase it as "the particle's spin is not measured. Here are the probabilities that it may be +1/2: (math); and that it may be -1/2: (math)." I think this simple choice of phrase makes quantum mechanics seem a lot less silly. But, if you want to say "the particle is both spinning up and not spinning up," (or, "the cat is both alive and dead"), you can very well phrase it that way. It's still silly. Nimur (talk) 16:08, 28 June 2011 (UTC)[reply]
Who said QM is not supposed to sound silly? If the electron is in a quantum superposition of a spin up and a spin down state than it is truly both at the same time. Dauto (talk) 16:15, 28 June 2011 (UTC)[reply]
You guys are all basing your statements on only one of the many possible Interpretations of quantum mechanics, specifically the common Copenhagen interpretation, which deals with such things as things existing in multiple states simultaneously, and with observation causing "collapse of the wavefunction" to produce the result; that is that the observation itself causes the result, somewhat non-mechanisticly. There are other, perhaps equally valid interpretations, such as the Ensemble interpretation, which takes a more agnostic view on the connection between quantum mechanics and reality. The mental gymnastics which is sometimes required (such as our simultaneously alive-and-dead cat) to make the Copenhagen interpretation work is philosophically unsatisfying. It is perhaps really much better to just state that our knowledge of the cat's living/dead state is incomplete without observation. In any instant, it may be alive or it may be dead, but it will be definately one or the other of them, irrespective of our observation. What changes is which variables we feed into our equations to decide if it is alive or dead; once we observe the cat. The distinction is deciding whether or not the event occurs only at the moment of observation or at some impossible-to-determine time before the observation. While philosophically the second proposition feels better, the first proposition actually makes the mathematics much simpler, since it gives us a specific point in time to determine when the "event" occurs; since it doesn't actually matter "when" prior to our observation, and since it is literally impossible to know "when" without observing it (i.e. you literally cannot determine the living/dead state of the unobserved cat unambigously from first principles, you can ONLY determine it empirically via observation), the first proposition works better for the purposes of making the math work. The end result is it doesn't really matter (from a scientific point of view) which "interpretation" works better for you, philosophically (that's why these are "interpretations" and not theories or laws or models, they don't require rigourous proof), the equations yield the same results either way, which is just the probability of the cat being dead as a function of time, and nothing more. What quantum mechanics does is destroy our belief in the "clockwork universe", that somehow given a perfect knowledge of initial conditions, one can deterministicly predict exact outcomes of any event before it happens. QM clearly shows this to be not true; one can only predict the probability of any event happening; even perfect knowledge of initial conditions cannot, for example, predict the exact time when a single discreet particle will decay. --Jayron32 16:48, 28 June 2011 (UTC)[reply]
Bohr didn't believe in wavefunction collapse as a physical process, so to the extent that "Copenhagen interpretation" means "Bohr's interpretation", the Copenhagen interpretation doesn't have wavefunction collapse in it. I don't think he believed in physical superpositions either.
The testable predictions of quantum mechanics are probabilities of the form . This is the probability that if you prepare a system in the state and subject it to a physical environment described by , you'll then find it to be in the state . The idea of wavefunction evolution and collapse is that, after you prepare the wavefunction in state , it changes gradually into , which is generally not the same state as . This mismatch is resolved by having the wavefunction suddenly, discontinuously jump from to at the moment of the measurement. There's no evidence that this happens; it's not experimentally testable. You could just as well say that the wavefunction suddenly "decollapses" to the state immediately after the preparation, then gradually evolves from there to . This kind of arbitrariness is the reason it's problematic to take an evolving wave function as the ultimate physical reality. -- BenRG (talk) 06:00, 29 June 2011 (UTC)[reply]
Example: Deutsch' thought experiment involving superpositions of entire observers observing different outcomes. Here one considers observing the z-component of a spin that is polarized in the x-direction. If the whole system of observer plus spin is isolated from the environment, the entire system will be a superposition of the two outcomes. Then the observer in each branch forgets, in a reversible way, the outcome of the measurement, by dumping the information back on the spin. That then restores the spin to its initial state. The observer, however, does keep the information that a measurement of the z-component was carried out.
The observer can then verify that the spin is polarized in the x-direction after this procedure, even though he remembers measuring the z-component. This then proves that when the z-component was measured, both sectors in which different outcomes were obtained really exist (if only one sector really exists after measurement, the spin would not be restored to its original state). Count Iblis (talk) 18:00, 28 June 2011 (UTC)[reply]


Another example: Quantum mechanics in a timeless universe.
Many theoretical physicists believe that time doesn't exist (my opinion is that those that do think time exists mostly haven't thought this issue through to its logical conclusion). Then the world today can, in this interpretation, be said to exist in the early universe as a complicated non-local superposition. Count Iblis (talk) 18:28, 28 June 2011 (UTC)[reply]
Jayron, you can interpret QM however you want. That will not change the fact that, as Count Iblis pointed out, |a'> = 1/sqrt(2) [|a> + |b>] is also a possible state so superposition of states is allowed. This has been confirmed by experiment ad nauseam. Dauto (talk) 01:00, 29 June 2011 (UTC)[reply]
Nothing I said has ever denied that. I, infact, fully endorse that, and I will again thank you for agreeing with me. The equation noted by Count Iblis is true, and doesn't need me to agree with it to be true. I don't see where I ever said it was wrong, however. The real existance of superposition of states is real and really exists. I don't know how many more ways I can say that to make you think that I agree with it, which I do. (and again, my agreement is not needed for its real truth to be really true). I should also note that I don't interpret quantum mechanics in any manner. I don't have any particular wants about how it is interpretted, and have nothing to add to existing scholarship on the issue. --Jayron32 01:04, 29 June 2011 (UTC)[reply]
Quantum logic may be relevant to this question, but I'm not sure since I never grokked it. -- BenRG (talk) 06:00, 29 June 2011 (UTC)[reply]
See also duality. QS supposes that an observer is required to mediate between the two outcomes and break the symmetry of duality, and until then both outcomes are realized. ~AH1 (discuss!) 01:18, 30 June 2011 (UTC)[reply]

MRI question

Would an MRI be better at finding bullet fragments then an xray? Googlemeister (talk) 15:45, 28 June 2011 (UTC)[reply]

You probably need to define "better." MRIs are bulkier, more expensive, and harder to transport than a standard x-ray radiograph imager. Nimur (talk) 15:47, 28 June 2011 (UTC)[reply]
There are risks with taking MRIs of bodies containing magnetic metals. Even though bullets are typically made of non-metallic lead, there may be fragments of magnetic metal from the shell casing, and steel ammunition is sometimes used (e.g. in shotgun pellets). Studies show that the metal fragments are unlikely to move much, but there is a risk of damage to internal organs.[11][12] --Colapeninsula (talk) 15:55, 28 June 2011 (UTC)[reply]
"Non-metallic lead" ? I assume you mean "non-magnetic". StuRat (talk) 16:10, 28 June 2011 (UTC)[reply]
Better as in more likely to find the bullet fragments, the bulkyness of an MRI vs an xray machine is probably not of paramount importance (unless you are talking about military field applications I suppose) since both are too large to fit into your standard ambulance and cost usually isn't the biggest concern of someone with a bullet in them. I did not consider that some ammunition was steel. I guess I need to read up on that. Googlemeister (talk) 16:07, 28 June 2011 (UTC)[reply]
I would expect bullet fragments to show up quite nicely on an X-ray, so no need to resort to the more expensive MRI. StuRat (talk) 16:11, 28 June 2011 (UTC)[reply]
To expand on StuRat's statement, I would expect that (ignoring all other factors such as cost and complicatedness of use, complications due to magnets, and other issues) that an X-ray would be equally as good as an MRI in finding bullet fragments, that is once the image is obtained, a trained professional should have no more trouble finding a bullet in the image with an X-ray image than with an MRI image. If we include the other issues already noted above, the X-ray is probably better than an MRI for this application. --Jayron32 16:14, 28 June 2011 (UTC)[reply]
Here's a useful link: a research paper that performed comparative virtual autopsy using MRI and CT, and comparing them to the X-ray radiography: Image-guided virtual autopsy findings of gunshot victims performed with multi-slice computed tomography (MSCT) and magnetic resonance imaging (MRI) and subsequent correlation between radiology and autopsy finding (2003). Nimur (talk) 16:17, 28 June 2011 (UTC)[reply]
I think there needs to be some clarification on your use of the term "x-ray" -- CT scan machines emit ionizing radiation, as do conventional x-ray machines, so what exactly would be your criteria for separating the two? Projectional x-ray images (such as a chest x-ray or a dental panoramic radiograph) would not locate the bullet fragments in 3 dimensions, making localization of the fragments for surgical removal difficult. A full body CT scan (or CT scan of that portion of the body known to possess the fragments would allow the practitioner to see the fragments in an axial slice. Whether or not MRI would be better or worse, though, I cannot say, because (as a dentist) I'm completely unfamiliar with MRIs. DRosenbach (Talk | Contribs) 17:00, 28 June 2011 (UTC)[reply]
With traditional X-rays, they take two, at 90 degree angles, to locate objects in 3D. For example, one from the front and one from the side. StuRat (talk) 17:43, 28 June 2011 (UTC)[reply]
Actually MRIs are very bad for finding metal. Metal objects don't even show up on an MRI; they distort the image in the area around them, though, making it sometimes possible to deduce their presence. Also if the metal is magnetizable (lead is not), the magnetic fields in an MRI will exert a strong force on it, which is obviously not a very good thing for a bullet fragment. CTs are absolutely perfect for finding metal, even tiny pieces just leap out at you if you look at one. Looie496 (talk) 02:10, 29 June 2011 (UTC)[reply]
I was just about to say the same thing, but Looie beat me to it. Panel (b) of the figure on this page shows an MR image of a brain with a bullet lodged inside. The bullet's distortion of the magnetic field causes the dark 'hole' and the adjacent bright 'flare'; you'd see something similar with any metallic object. Not only does it make it very difficult to precisely locate the bullet itself, but it also conceals and distorts the appearance of nearby anatomical structures. With steel (or other ferromagnetic) shrapnel, pellets, or bullet fragments, one also has the additional risk of local heating and migration of the metal item in the magnetic field, which may cause pain or injury. TenOfAllTrades(talk) 02:39, 29 June 2011 (UTC)[reply]

Multiple Alleles

Hello. If the dominance hierarchy of coat colour in Netherland Dwarfs is C (dark gray) > cch (chinchilla) > ch (light gray) > c (albino), why do cchcch and cchc code for light gray instead of chinchilla and why do chch and chc code for point restricted instead of light gray? Thanks in advance. --Mayfare (talk) 17:36, 28 June 2011 (UTC)[reply]

Because binary genetics (even multiple allele binary genetics) is rarely useful for anything more than pedagogical purposes; that is it is helpful in demonstrating the basic principle, but in actual practice, the physical traits expressed by genes are often the result of complex effects, sometimes in the second or third order (for example, the gene doesn't directly code for the structure of the pigment, it codes for the structure of a component of the pigment, or it codes for the structure of a substance that is involved in the manufacture of the pigment some several steps back from the pigment itself). In particular, broad traits such as pigmentation represent what are called polygenic traits which are coded by multiple genes. For some analogs in humans, see Eye_color#Genetics and List_of_human_hair_color_genes, which lists for humans some 12 genes responsible for hair color; you'll also note that many of these genes code for things OTHER than hair color as well, that is there are not only multiple genes which determine hair color, there are multiple traits coded for by each gene (of course, depending on how you define "trait"). I would expect the genetics of other mammals to be similarly complex. --Jayron32 19:09, 28 June 2011 (UTC)[reply]
I took the liberty of linking Netherland Dwarf above, as I didn't know it was a rabbit. ;) (Oryctolagus cuniculus to be precise) Here's a public access paper about the E (extension) locus and a similar dominance sequence,[13] but I'm still hunting... alright, finally got someone to say that the c (albino) locus is indeed tyrosinase in O. cuniculus[14] (never take that sort of thing for granted...) chinchilla and albino alleles are indeed due to mutations in the CDS (compared to C (dark gray). Which leaves light gray... didn't find that; there's only one sequence coming up in NCBI, namely [15]. I even ran a BLAST search and came up with no other nucleotide sequences. It looks like the relevant work may be so old it isn't indexed - indeed, perhaps the sequence was never determined. This is generally the point where you have to pick up a phone and call someone on the last paper to come out about it in the hope of getting some answers, which I'm not going to do now for this exercise; my guess is that we're looking at some very old data. So I can't say exactly why these alleles would have a more peculiar dominance pattern. Also, the data you describe sounds unlikely - I'd expect two recessive alleles of the same type to deliver the advertised phenotype! Hmmm... for further information see [16] - it's apparent there is a lot more breeding of rabbits going on than I'd ever imagined! ;) And yes, two chinchilla alleles can lead to a light gray color, though there's some lingering confusion on the point. Hmmm, so much data and so few sequences in NCBI :( Wnt (talk) 23:19, 28 June 2011 (UTC)[reply]

From the Caspian Sea to the Black Sea

I included two questions in a posting above, and one of them got no attention, so here it is alone.

Is there any truth in Leonardo da Vinci's statement that there are subterranean rivers flowing from the Caspian Sea to the Black Sea? Michael Hardy (talk) 20:49, 28 June 2011 (UTC)[reply]

That seem highly unlikely as the Caspian Sea is actually 28 metres lower than the Black Sea and there is the Caucasus mountains in between. Any river flow would be from the mountains of the Greater or Lesser Caucasus ranges into one or the other seas. Mikenorton (talk) 21:17, 28 June 2011 (UTC)[reply]
As an aside, there has been means of getting between the seas for quite some time. The Volga–Don Canal goes back several hundred years; and since the Volga and Don are relatively close (100 km or so), that makes portage between the rivers (and thus between the Caspian and Black seas) reasonable even before the canal was built. However, this has nothing to do with natural river flows between the seas; as noted water generally doesn't flow uphill. --Jayron32 23:24, 28 June 2011 (UTC)[reply]
The Volga–Don Canal seems to go back to 1952 except for a couple years in the early 1700s. Rmhermen (talk) 02:55, 29 June 2011 (UTC)[reply]
You may also be interested in Lake Manych-Gudilo, Paratethys and Black Sea deluge theory. The salinity of the entire region is explained by previous sea level rise (25+ metres), though not necessarily by any subterranean rivers. ~AH1 (discuss!) 01:15, 30 June 2011 (UTC)[reply]

Are we evolving?

File:Skeleton of a man of the future.svg
We are evolving. Into this.

Is the human species evolving? Indeed, since modern medicine is able to keep alive humans who would die under other circumstances, then, we are indeed getting weaker. Wikiweek (talk) 21:49, 28 June 2011 (UTC)[reply]

As long as some people die earlier, and reproduce less, than others, evolution will continue. It might possibly slow down or evolve in a different direction (say favoring people genetically predisposed to use seat belts), but it won't stop. StuRat (talk) 21:52, 28 June 2011 (UTC)[reply]
Evolution is just gene flow over time. That happens just by living and reproducing and so forth. If you mean, "are less favorable traits being selected for over time?" (e.g. dysgenics), it really becomes a question of what one defines as "less favorable." It's a pretty subjective measurement and the level of subjectivity becomes clear when you consider some traits that are widely shared and found to not be such a big deal, but would have been a significant hindrance (or even fatal) a long time ago, such as poor eyesight or appendicitis. In the case of IQ, there is evidence that has only risen over time (see Flynn effect). If by "weaker" we mean only "that they would have not survived without modern medicine," we can only then ask why we should take that as a meaningful criteria. There are lots of people who wouldn't have survived without modern morality or social institutions, in general. What does that tell us except how lousy the past was?
It's also a fallacy to consider modern medicine somehow "unnatural" to the species. It's as natural as anything else we've been up to. It's new (from a species-long perspective), but that hardly makes it any less "natural". --Mr.98 (talk) 21:59, 28 June 2011 (UTC)[reply]
Would the Flynn effect imply that intelligent people have more offspring than the stupid ones? Otherwise, it might be a non-genetic effect. — Preceding unsigned comment added by 88.9.106.0 (talk) 10:49, 29 June 2011 (UTC)[reply]
Define "weaker". Without "modern medicine" Stephen Hawking would have undoubtedly died from his disease long before he made many contributions to our understanding of how the universe works. Insofar as an improvement and refinement to the canon of human knowledge is generally a Good Thing, modern medicine has made people stronger. Also, there is the problem with defining humans as somehow "extranatural", that somehow what happens to us or is done by us is somehow outside of "nature", whatever that means. We're particularly good at altering our environment and ourselves in ways that benefit us (at least in the short term), but we're not unique in that regard; every other living thing constantly does that. We alter the environment in human ways, but that's not necessarily a meaningful statement; rabbits alter the environment in lupine ways, and trees alter the environment in dendritic ways. We're active parts of the system, not passive managers or observers of it... --Jayron32 23:20, 28 June 2011 (UTC)[reply]
Yes, I like to refer to "artificial" as a subset of "natural", as anything man-made is indirectly made by nature. StuRat (talk) 23:24, 28 June 2011 (UTC)[reply]
It's true that saving people who would have died frustrates one small step of natural selection - but natural selection, like death, is patient, and sooner or later - unless we fix such genes ourselves - it will drink its fill. Many of the genes we possess are optimized for conditions we don't expect to experience. Consider, to begin with, how remarkable it is that a species from Africa can survive exposure of any part of its skin to the bitter cold of the Arctic - because there have been some very cold years indeed, long ago. Mutations like the one in CCR5 were probably kept by our ancestors for thousands, maybe even millions of years, as a distant memory of other HIV-like outbreaks before this one - and, most likely, the same is even more true of the normal variant most people carry, against some other disease not yet identified.
In order to make good decisions about genetic therapy to compensate for the loss of natural selection, we should recognize that alleles arise at some point in history, and that while some are ancient and valuable, others are recent and have no special historical significance. Many of the cruelest genetic diseases are of the latter sort, arising in the person affected, or some recent ancestor. If we resolve to preserve the range of genetic diversity we inherited from people 1000 or 10000 years ago, while resolutely putting right any recent mutations that cause noticeable trouble, we can maintain our vital genetic diversity and human essence, permit the free flow of speculative evolution into the future, and yet counteract disease more effectively than if people died from it. Wnt (talk) 00:18, 29 June 2011 (UTC)[reply]
Agreed, and even if we do "fix such genes ourselves", that will probably fall to a different kind of natural selection. People's preferences as to what genes are desirable, and which are not, will change over time to fit current conditions and cause evolution that way. APL (talk) 03:00, 29 June 2011 (UTC)[reply]
How about a reference, here on the Reference Desk? Our article Human evolution includes the section Recent and current human evolution. Admittedly it is short, and I was surprised not to see a large article on this topic. PS: I have added an illustration of the creature into which we are evolving, according to a Russian scientist. Comet Tuttle (talk) 03:40, 29 June 2011 (UTC)[reply]
Man, that guy must have a strong neck. APL (talk) 06:46, 29 June 2011 (UTC)[reply]
Before we jump to conclusions that the skeleton of the Star Trek-like alien is the future of mankind, ask yourself this: Which segment of the population is growing faster? The ones with the really "big brains"? Or "everyone else"? ←Baseball Bugs What's up, Doc? carrots16:33, 29 June 2011 (UTC)[reply]
Good point. What do you think is the correct answer to that question?
Here are some books that look at this question: Future Evolution by Peter Ward and Man After Man by Dougal Dixon. Both are good reads, Future Evolution is pretty dry and scientific, while Man after Man is really pure fantasy couched with scientific terms, but I found it fascinating nonetheless (although it wasn't as good as Dixon's other book After Man and unfortunately both of Dixon's books are rare and expensive). --Daniel 17:35, 29 June 2011 (UTC)[reply]
The people with the big brains reproduce much slower then everyone else, but when the formula for perpetual youth is discovered, then how many of the 12 kid families who live in the trailer park will be able to afford the youth elixer? Googlemeister (talk) 18:55, 29 June 2011 (UTC)[reply]
That is a good point, we can't take a current pattern that has only existed for a couple hundred years at most and assume it will continue for a million. Although Mike Judge's film Idiocracy imagines it continuing for a few hundred more. Even still, I don't think the trend is actually less intelligent people having more children, it is poorer people. --Daniel 19:02, 29 June 2011 (UTC)[reply]
I've seen this argument made, except... isn't it a general social law that the harder someone works, the less he makes? The poor people who breed the most may be undereducated and underprivileged, but I suspect in a truly fair assessment they are much more useful to themselves, their children, and humanity than all the wealthy CEOs and officials. In the long run, a humanity which replicates the qualities of the poor is sure to survive, but one based only on the rich would surely succumb to general madness. Wnt (talk) 23:19, 29 June 2011 (UTC)[reply]
Do people that live in trailer parks have smaller than average brains? I don't think so. I think I sense some kind of underground prejudice bubbling up to the surface. In fact, out of those 12 kids, the one with the largest brain might be the one that finds a better life while his "normal"-sized-brain siblings might end up in jail or dead. We don't know, but it is possible that trailer parks is exactly the place where evolution of larger brains is happening right now as we "speak". Dauto (talk) 20:43, 29 June 2011 (UTC)[reply]
Neanderthal had a larger brain then humans. Any guess on how well it turned out for them? Googlemeister (talk) 21:00, 29 June 2011 (UTC)[reply]
And whales have even bigger brains... But what does that have to do with human evolution? Dauto (talk) 21:58, 29 June 2011 (UTC)[reply]

Does sexual selection have anything to do with this? What I mean is that during hundreds of millions of years of evolution, animals have had to deal with fluctuations in selection pressure. If animals become tame too soon after predators vanish, they would have problems when the predators return. This problem could then lead to a selection pressure for sexual selection to lead to "survival of the fittest", even if you don't need to be fit to survive. Count Iblis (talk) 23:37, 29 June 2011 (UTC)[reply]

The number of young peoples deaths from firearms in America is high enough to exert a significant evolutionary pressure. I've been wondering if it will lead to better coping with the damage or a psychology to avoid the situations in the first place or what will happen. By the way the people with less children tend to be middle class rather than the rich or poor. Dmcq (talk) 00:20, 30 June 2011 (UTC)[reply]
Can you in any way back up your first assertion? --Tagishsimon (talk) 01:19, 30 June 2011 (UTC)[reply]
It's fairly well known I believe that one in a thousand difference in reproduction is enough to overcome genetic drift and exert evolutionary pressure and there seems to be something like 5 in a thousand such die from gunshot in the States I think. Car accidents would be in there too though not such a high proportion of young people are killed. Dmcq (talk) 08:20, 30 June 2011 (UTC)[reply]

Deep Tissue Massage

I was looking at the website of a day spa and one service they called "Deep Tissue Massage" which they described as "Is a type of massage therapy that focuses on realigning deeper layers of muscles and connective tissue." Realigning layers of muscle and connective tissue sounds like a bogus claim but if anyone has proof that it can really happen by one human's manual kneading I'd like to know.198.228.193.74 (talk) 22:17, 28 June 2011 (UTC)[reply]

Perhaps it helps just by stretching those muscles and connective tissue a bit. Even if those particular claims sound questionable, it still seems worthwhile, whether for psychological/stress-reduction/relaxation reasons or something else. StuRat (talk) 22:42, 28 June 2011 (UTC)[reply]
At $85 for 50 minutes, I'm pretty sure it couldn't make me feel good enough not to experience Buyer's remorse, but of course that's just me, and to each his own.198.228.193.74 (talk) 22:49, 28 June 2011 (UTC)[reply]
Oh, and I forgot to mention the primary benefit, working out all those farts inside you. Now you see why they charge so much. :-) StuRat (talk) 23:09, 28 June 2011 (UTC) [reply]
Another good one is "Reiki Healing," which says: "Japanese technique for stress reduction and relaxation that also promotes healing. It is administered by "laying on hands" and is based on the idea that an unseen "life force energy" flows through us and is what causes us to be alive" 70 bucks for 50 minutes. I'm in the wrong business.198.228.193.74 (talk) 10:54, 29 June 2011 (UTC)[reply]
There's probably also a keen distinction between "I do XXX because of the way it makes me feel." vs. "I do XXX because it makes me healthier" here. As long as XXX doesn't otherwise harm you, there's probably nothing wrong with "doing it because it makes you feel better", which as far as I am concerned, is a positive enough outcome even if outlandish health claims aren't backed up by hard science. --Jayron32 23:14, 28 June 2011 (UTC)[reply]

(But is it enough to trust the practitioner who makes the claims? I wonder this about chiropractics. Most of those chiropractic places have big banners advertising the health benefits of "laser massage" and other obvious flim-flam. Why would you let such a snake-oil salesman near your spine?) APL (talk) 02:04, 29 June 2011 (UTC)[reply]
The key phrase is "as long as XXX doesn't otherwise harm you". Its an important concept, and you should re-read my statement keeping that phrase in the forefront of your understanding of it. It is a conditional statement, and removing it from my writing above changes the meaning of it. It appears from your response that you inadvertantly removed that phrase when reading my writing, or ignored it, or otherwise didn't notice that I wrote it, so I thought it prudent to bring it to your attention again to assure that its full weight and meaning did not go unheeded. --Jayron32 02:39, 29 June 2011 (UTC)[reply]
Oh yea, I noticed it. And I wasn't trying to refute you or argue with you. I agree completely. I was just thinking 'out loud'. I was thinking of a situation where procedure XXX is known safe or low risk, but the local practitioners appear to be flim-flam men making insane claims. Since you can't actually know for sure if something will "otherwise harm you" before you do it, and afterwards the question is kind of academic. APL (talk) 02:56, 29 June 2011 (UTC)[reply]
Trudat. --Jayron32 03:04, 29 June 2011 (UTC)[reply]

You might like to follow up this site's reference. We also have an article on it: deep tissue massage. You might also like to look at Swedish massage. --TammyMoet (talk) 07:42, 29 June 2011 (UTC)[reply]

June 29

Isn't hair loss an evolutionary trait?

Is it really a health problem? I've no doubt that were it not for sexual selection, we could easily become hairless in the future. (And in case you're wondering, I've beautiful hair.) Imagine Reason (talk) 01:04, 29 June 2011 (UTC)[reply]

I think you could say that for any trait that's genetic or otherwise hereditary.
I imagine it's a very small comfort to the afflicted, though. APL (talk) 02:01, 29 June 2011 (UTC)[reply]
Is there any decent study of the different rates of hair loss among different human populations. As a mature Australian of northern European ancestry, my shiny dome seems to be becoming larger by the day, but all my Australian Aboriginal friends of similar age still have masses of wavy locks. (It seems they use the same silver hair dye as me though ;-) ) My theory is that, in an evolutionary sense, keeping one's hair for longer mattered more in the Australian sun than in the Scottish mist. Anyone got any better information than my totally original research on this small sample? HiLo48 (talk) 02:34, 29 June 2011 (UTC)[reply]
Our Baldness article does touch on a couple of studies that measured people's perception of the bald and balding, on the theory that this perception may serve a social purpose for the chrome-domed, and hence evolution and natural selection may have encouraged it. For you, that is. Comet Tuttle (talk) 03:36, 29 June 2011 (UTC)[reply]
Somewhere there is a silverback gorilla who is just aching to have access to Just for Men. --Mr.98 (talk) 03:42, 29 June 2011 (UTC)[reply]
Unexpected hair loss can be a problem, i.e. a sign of illness of some kind. However, if some of your ancestors went bald, that would increase your odds of going bald due to genetics. ←Baseball Bugs What's up, Doc? carrots04:42, 29 June 2011 (UTC)[reply]
I don't know that it is a "health problem" on par with say, cancer, but it certainly can be a "quality of life" issue; if a person feels insecure about their hair loss, it can affect their quality of life in negative ways, and this is a real problem for people who are balding (and who care, I should add. There are, of course, balding people who don't give a shit). Insofar as people feel that their quality of life is diminished by hair loss, then its something that will be dealt with. Insofar as it being an "evolutionary trait", sure hair loss is an evolutionary trait, but confirming that fact is akin to confirming the fact that "green is a color" or "17 is a Prime Number". Its true, but I'm not sure that being true implies that it is also important, insofar as being bald is something that is a driving force for natural selection. Some traits are preserved in the genetic history not merely because they are advantageous, but just because they aren't disadvantageous enough to cause it never to be passed on. Since most people don't become bald until after their usually mating age (i.e. most people have their kids in their 20s and early 30s, but baldness doesn't often strike until a few years after that) would be a small bit of evidence in support of the "sexual selection" arguement, (i.e. sexual pressures push men to have a full head of hair, at least long enough to mate) though much of genetics is a complex, probabilistic situation and not strictly "X is good so exists, Y is bad so should never exist" sort of situation. Androgenic_alopecia#Hair_loss_and_genetics has some actual background on the genetic component of baldness. --Jayron32 05:46, 29 June 2011 (UTC)[reply]
Interesting link. Thanks. Imagine Reason (talk) 18:42, 29 June 2011 (UTC)[reply]
As I said in the evolution thread above, I am loath to call any common genetic variation a disease. Generally speaking, a gene variant which has existed for over tens of thousands of years, or longer, should have some merit. Now there are nuances to that where heterozygosity is involved - for example, I would argue that the sickle cell anemia gene should not be called a "disease gene" where malaria is endemic; yet sickle cell anemia is a disease; perhaps lack of that trait is also a disease when malaria is a leading cause of death - but if we can obliterate malaria from the face of the Earth, perhaps we could justly play God and say it is time for evolution to move on. But these nuances don't apply to baldness, where only one copy of the gene is affecting how men look. Any present fashion on the point is transient, without long-term significance. Otherwise... well, would you say that being black in Confederate America was a disease? Wnt (talk) 23:08, 29 June 2011 (UTC)[reply]

Antarctica without an ice sheet in the summertime?

I was watching a documentary about Antarctica a couple days ago and in it, it said that Antarctica is freezing, even in the summertime, because its ice sheet reflects the sunlight away from the land. But if Antarctica didn't have an ice sheet (and assuming the continent wouldn't be flooded from what I see from this image), approximately how hot would it be in the summertime, when the sun shines on the continent for half of the year? 64.229.6.52 (talk) 02:02, 29 June 2011 (UTC)[reply]

You can look to other locations on the antipodes from Antarctica which lack an ice sheet to get an idea. Barrow, Alaska has average daily temperatures about 10 degrees farenheit (5-6 degrees Celsius) over freezing. Tuktoyaktuk has similar temperatures. The closer you get to the poles, the colder it would get, even with several months of constant sun; the angle of insolation is so oblique that, though the sun is technically shining for months at a time, it provides very little heating. Imagine a ball thrown directly at a target, and then the same ball at the same velocity hitting the target with a very slight glancing blow. The amount of energy imparted to the target by the ball is very much tied to the angle it strikes the target at; its the same with the heating effect of the sun. BTW, you may also want to read albedo which is related to the initial presumptions about the effect of snow on the climate of Antarctica. --Jayron32 02:53, 29 June 2011 (UTC)[reply]
Should you really be using an analogy using momentum? Comet Tuttle (talk) 03:33, 29 June 2011 (UTC)[reply]
(edit conflict)Note that those figures for Barrow are the average high temperatures in the summer; the average temperature year-round is well below freezing. I suspect that the reflection of ice has some effect on the near-surface temperature; but not much, especially near the ocean. A good deal of the low temperatures in the coldest areas of Antarctica are due to it being as high as some of the highest mountains in the United States; due to adiabatic cooling, the higher you go above sea level, the colder it will be. Vostok Station, which had the coldest temperature ever recorded on the Earth's surface, is almost 3,500 metres (11,500 ft) above sea level, atop a giant ice sheet miles thick. Compare its mid-summer average temperature of (−32.1 °C (−25.8 °F)) to that of Alert, Nunavut, Canada (3.3 °C (37.9 °F)), which is further north, but is at sea level; this is almost the exact difference you would expect for an elevation change of 3.5 kilometers.-RunningOnBrains(talk) 03:37, 29 June 2011 (UTC)[reply]
West Antarctica without an ice sheet would be largely ocean, and this has happened as recently as the Eemian interglacial. The albedo effects of any vegetation would also be a factor. ~AH1 (discuss!) 01:11, 30 June 2011 (UTC)[reply]

Gases

1.many properties of gases (and other states of matter) are explained by the movement of the particles in them, but my problem is that I can't add them all together to have a general image of gas behavior. so my question is what is the difference between motions that make currence(like wind),thermal motions,mechanical motions that let the sound propagate in the gas,motions that cause the matter to make electromagnetic radiation(heat),etc.Iknow that some of them are probably the same but I want to clearify it.Thermal motions are described "random" but what is th definition of random anyway?and about mechanical waves, how can the random-moving particles suddenly oscilate in srtaight lines? is exactly that way?

2.another question I have is that if you have acloud of Ionoized gas(plasma) and make sound waves in it will the cloud propagate electromagnetic waves with exactly the same frequency because of its moving charged particles?thanks a alot.--Irrational number (talk) 07:04, 29 June 2011 (UTC)[reply]

I'm mostly unable to understand what you are trying to ask in #1, but as far as the definition of "random", see our article randomness. ~ Mesoderm (talk) 08:38, 29 June 2011 (UTC)[reply]

What I mean is what is the difference (if there is any difference) between microscopic motions that cause macroscopic phenomenon like I mentioned above (currents,heat,sound,radiation,etc).Because when I ask my teacher what is the cause of heat,sound and other effects the answer is always motions of microscopic particles.I hope you understood me (i'm not a native speaker of English, so I apologize!).Thanks in advance. — Preceding unsigned comment added by Irrational number (talkcontribs) 09:38, 29 June 2011 (UTC)[reply]

In the kinetic theory of gases the macroscopic properties of a gas (such as temperature and pressure) are related to the microscopic motions of its individual molecules. We cannot follow the individual motions of every molecule, so instead we assume these motions follow some given random distribution, and we relate the macroscopic properties to statistical averages (across time, space or population space) of these random motions. Usually the motions of the individual gas molecules are assumed to be distributed in such a way that the time average of the total momentum of the molecules is zero. But a constant wind could be modelled by adding a non-zero constant wind velocity on top of these random thermal motions, so as to give a non-zero time average. And a sound wave could be modelled by making one boundary (or wall) of the container vibrate with, say, a sinusoidal function of time, thus creating a pressure wave that propogates through the gas. In practice, it usually more feasible to study wave propogation by modelling the gas as a continuum and applying a differential equation such as the wave equation. But, in principle, all of the macroscopic phenomena that you mention could be deduced from the motions (and rotations) of individual gas molecules, if we knew them in sufficient detail. Gandalf61 (talk) 10:52, 29 June 2011 (UTC)[reply]
Try this on for size. Start with a gas with absolutely no non-random motion. That is, you have a perfectly "still" set of conditions. What you have in that case is all of the particles moving randomly, as individual particles, that is there are equal numbers of particles moving "left" and "right" so that, in the bulk material, their effect is to exactly cancel. Now, introduce a disturbance to this material, say in the form of a force acting on one end of the gas (in other words, "wind".) What happens now is that some larger fraction of the particles is now moving in one direction more often than the other; that is whereas if before 50% were moving left and 50% were moving right, say 60% are moving left and only 40% are moving right. Now you have wind. --Jayron32 18:41, 29 June 2011 (UTC)[reply]
Wave propagation works via the energy transfer from particle to particle of converted random motions, so that this energy has peaks and troughs in intensity that travel in a constant direction away from the source. ~AH1 (discuss!) 01:09, 30 June 2011 (UTC)[reply]

General Tau Theory

Is this real? I just looked at General Tau Theory and it seems to be nonsense to me. 78.25.219.244 (talk) 07:59, 29 June 2011 (UTC)[reply]

Looks like ramblings of a mad-man to me. Note that the one reference is a dead link and so is the external link to "Dave N. Lee". The one relevant internal link is James J. Gibson, which appears to be a proper article.
This might very well be some "new age" theory, but those don't have any scientific basis. We might remove the article, in that case, unless it's notable enough. That is, even the whacky theories of certain cults are allowed articles here, not because anyone believes that they are true, but because they are notable. StuRat (talk) 08:26, 29 June 2011 (UTC)[reply]
The criteria for "odd theories" is described at WP:FRINGE. --Mr.98 (talk) 13:47, 29 June 2011 (UTC)[reply]
Apparently, yes. [17][18][19][20][21] Provided that "real" includes a theory of motor coordination processes discussed by academics in the peer reviewed literature. Dragons flight (talk) 08:41, 29 June 2011 (UTC)[reply]
OK, in that case it needs a major rewrite, so it actually defines it's terms, like "tau" (right now it just has a link to a general discussion of the Greek letter). StuRat (talk) 13:57, 29 June 2011 (UTC)[reply]
I've fixed the links in the article - it seems to be genuine, or, at least, hosted on Edinburgh University's website. Scottish taxpayers' money gets spent on this sort of thing? Ah well... Tevildo (talk) 22:12, 29 June 2011 (UTC)[reply]
Two words define this sort of situation... "professor emeritus" I don't know why - I don't believe for a minute that it's truly an age-related effect. Rather, I think there is a complex social process by which people become marginalized and begin to seek perhaps overly innovative strategies. For that matter, I don't know if "rogue humans" can be defined biochemically. Wnt (talk) 23:12, 29 June 2011 (UTC)[reply]
All Gibsonian stuff tends to look weird to those who are unfamiliar with it, and this is no weirder than a lot of it. There don't seem to be any RS-level sources for this particular article, but Lee has a very extensive reputable publication record, including a (co-authored) paper in Nature. In short, there perhaps is not a proper basis for a Wikipedia article, but there is no call for this "ramblings of a madman" stuff. Looie496 (talk) 00:14, 30 June 2011 (UTC)[reply]
It seems to be a form of antireductionism, coupled with something related to the tau-function, which is an oscillation that seems to exponentially increase in amplitude. ~AH1 (discuss!) 01:06, 30 June 2011 (UTC)[reply]
No, it has nothing whatsoever to do with that. In Lee's version of motor control theory, tau is an estimate of the expected time until some event occurs. For example when a person swings a golf club, tau is the estimated time until the club comes in contact with the ball. When a person brakes a car to avoid hitting something, tau is the estimated time until collision given the current velocity (I think). Looie496 (talk) 01:38, 30 June 2011 (UTC)[reply]

50%t

What is %t?Curb Chain (talk) 19:23, 29 June 2011 (UTC)[reply]

A typographical error? The "t" is directly under the "5" key on most English language keyboards, which could easily lead someone to accidently depress it when typing "shift-5" to make the % sign. It should read "a relative humidity less than 50%" --Jayron32 19:29, 29 June 2011 (UTC)[reply]
Looking at the linked source, it shouldn't make any mention of humidity because the source only discusses temperature. -- kainaw 19:30, 29 June 2011 (UTC)[reply]

fixed. Dauto (talk) 20:27, 29 June 2011 (UTC)[reply]

potassium and sodium in seawater

Earth's crust contains 2.6% potassium and 1.8% of sodium so this roughly the same amount. But seawater contains 0.39 g/L potassium and 10.8 g/L sodium. This is a large difference. Both elements form nearly only water soluble salts which readily dissolve in water. Only the more complex silicates retain both elements. .I read somewhere that clay minerals retain potassium better than sodium, but is this effect enough to create this large difference in seawater? A nice book as a reference would be very nice. Thanks!--Stone (talk) 21:04, 29 June 2011 (UTC)[reply]

Low-gear torque

Hello. How can I prove that lower gears yield more torque than higher gears? Thanks in advance. --Mayfare (talk) 22:57, 29 June 2011 (UTC)[reply]

Probably by using the magnitude formula specified in the torque article. --Tagishsimon (talk) 23:07, 29 June 2011 (UTC)[reply]
This looks like a homework question. We don't do people's homework for them. However, here are some thoughts to start you thinking. A gear train can transmit energy, but it can't store energy. Therefore the work done on the input shaft must be equal to the work done on the output shaft, but the two will be rotating in opposite directions so the resultant work will be zero. When a torque T rotates through an angle A (measured in radians) the work done is TA. So the work done on the input shaft is TiAi and the work done on the output shaft is ToAo. The ratio of angles through which the input and output shafts move is related to the ratio of the number of teeth on each gear. Use this information to examine what the relationship might be between the torques on the two gears. Dolphin (t) 08:29, 30 June 2011 (UTC)[reply]

June 30

The universe, consciousness and the anthropic principle

Hi. Can a property of the universe be said in the following manner? This is not meant to advertise some cranky or crackpotting idea but just for discussion although the following ideas may induce semantic headache. Consider the following.

The product of human consciousness and sapience has been an anthropogenic proliferation in tools, eventually evolving to higher forms of technology, including artificial intelligence. This can potentially be characterized as a localized decrease in entropy, in which energy of chaos is re-positioned to form ordered substances. Although the second law of thermodynamics dictates that entropy must increase over time, localized increases in order via the self-ordering nature of matter are allowed, which is accelerated when consciousness is present.

Given the Big Bang theory of the formation of the universe, matter and antimatter were composed of high energies, infinite temperature and a primordial soup. After this initial energy forced inflation (cosmology) at rates approaching Plank velocities, the matter-antimatter asymmetry allowed the prevailing of matter over antimatter through the help of bosons and other factors. The matter assembled into stars, which later formed planetary systems and were components of quasars and later galaxies, which in turn became ordered into superclusters in bubble-like forms. Complex molecules arose on Earth, eventually forming into prokaryotes, then into complex life, which eventually evolved to humans today, which carry a form of conscious self-awareness.

In this context, the universe over time becomes more and more ordered, and although all matter such as stars eventually explode, the resulting debris forms into new stars and new order. Since both humans and the universe share this locally accelerated ordering capacity, could it be said that the Universe is dually conscious?

Alternatively, does the hypothesized supersymmetry breaking and multiverse scenario more than make up for this local decrease in entropy? Considering a multiverse, supose that its consistuents are order and chaos. Similar to the matter-antimatter duality within our universe, the scenario could either unfold where order prevails (a universe forms), or chaos does (a nearby universe is destroyed). Thus, would the passage of time result in the formation of more universes, so that the total heat contained in all universes increases over time, thus increasing total entropy on a quantum level? Or, can something about dark matter change this effect?

The anthropic principle proposes an almost infinitely long list of requirements for life to exist on Earth, so that the chance of such a universe existing is one over infinity. However, this is easily resolved given a multiverse, in which an infinite number of universes is created and destroyed. Of course, we don't know whether this is the correct infinity.

However, humans have succeeded in creating the basic building blocks of life. If the universe and humans are both conscious, depending on how consciousness is defined, could the existence of conscious life be considered a fractal subset of the macro-scale universe's consciousness? Or, would this only be true in retrospect, as the existence of conscious life itself is required for any realization of the anthropic principle?

Is time thus a non-material dimension, one that results in the conversion between matter and energy to cause the diversion of macro-scale and micro-scale entropies? There is also a human sense of time, which is potentially distinct from the actual flow of time relative to the conscious observer. Can this somehow make sense of the aforementioned context, provided that simultaneity is relative based on different observers in spacetime, so that each observer is positioned so that all other observers' futures in spacetime already exist?

Does any of this make meaningful sense? Thanks. ~AH1 (discuss!) 00:36, 30 June 2011 (UTC)[reply]

Not really, no. There's no evidence that the universe is conscious. The final few paragraphs look more like rubbing a set of random concepts together than anything else. --Tagishsimon (talk) 00:42, 30 June 2011 (UTC)[reply]
(edit conflict)I don't have the mental energy right now to fully wrap my mind around your whole argument, and you touch on a large number of highly-speculative and non-falsifiable topics (thus your whole argument here may be more philosophy than science), but I do have a few points:
  1. You make the assertion that human consciousness is "accelerating" a local decrease in entropy (increase in "order") on Earth which you take as truth; all of this without citation. Now, this may be true, but it would be nice to see some quantification of this before I take it as fact. We may be decreasing entropy on earth by refining metals from ores and extracting other pure materials from nature, but we are doing so primarily through the combustion of fossil fuels, which is a process which results in an overall increase in entropy.
  2. Even using "renewable energy" such as solar power, wind power, and geothermal power, this energy is ultimately all derived from solar fusion from the Sun, a process which results in a huge increase in entropy.
  3. You make the further assertion that "the universe over time becomes more and more ordered". This is demonstrably untrue: as I say above, fusion in stars results in an enormous increase in entropy as primordial hydrogen is fused into helium on astronomical scales. If I had some more time I'd do some scale-analysis to give some rough numbers, but it is safe to say it is enormously beyond the scale of any potential local decrease in entropy by humans (which, as I've said above, may not even be true).
  4. I'm no good on the philosophy front, but I do believe that most definitions of consciousness at least require a conscious entity to present the appearance of self-determinism; the Universe as a whole does not appear to demonstrate this feature.
  5. Almost infinity is not even close to infinity; the two are not even comparable. The odds of the existence of a universe where humans can live can not be zero because it does exist.-RunningOnBrains(talk) 01:08, 30 June 2011 (UTC)[reply]
  6. You also seem to be trying to think of the anthropic principle as a causal mechanism; this is nonsensical, as the anthropic principle is essentially answering the question of "Why are we here?" with "Because we are here." This is an over-simplification of the topic; the anthropic principle is really the only branch of philosophy I've done serious reading and thinking on. It is my favorite principle, since it really allows us to avoid the need to ask the question of why we are here. It helps me sleep better at night.-RunningOnBrains(talk) 01:16, 30 June 2011 (UTC)[reply]
On the issue of conscious entities lowering entropy: this dos not happen. If you clean up your desk, putting things that were initially randomly distributed, in some neat order, then the information needed to specify the initially disordered state is not lost. You have to act on that intial state to get to the final state, so the information about the initial state ends up in your brain. So, as your desk becomes more and more neatly ordered, your brain accumulates more and more random information. You can later forget about what you exactly did to clean up your desk, but then the information gets dumped into the environment. Count Iblis (talk) 01:17, 30 June 2011 (UTC)[reply]
Supposing the universe had some form of consciousness, could we somehow potentially test for it? The final paragraphs supposed that if the universe was conscious, then conscious life could be a fractal-like microcosm of the consciousness, or not. ~AH1 (discuss!) 01:51, 30 June 2011 (UTC)[reply]
Entropy, as a quantified theory, is defined on the molecular and atomic scale. Reordering macro objects will not result in a quantifiable decrease in entropy, though it may intuitively seem so to our consciousness. Calling it "localized" implies that the volume in question is less than typical entropy measurements, which is not the case. Keep in mind that moving objects, and even thinking of moving objects, increases entropy by converting food to more basic molecules (much like the fossil fuel reference mentioned). Mamyles (talk) 02:59, 30 June 2011 (UTC)[reply]
More importantly, it increases entropy because it decreases free energy; some food energy is always lost as heating the environment in addition to making muscles move; and that environmental heating is also an increase in entropy. --Jayron32 03:10, 30 June 2011 (UTC)[reply]
There's a common fallacy which is the notion that the complex can't naturally arise from the simple. It's often evoked for instance by proponents of intelligent design. They argue that the world is complicated and beautiful, which proves that it can't adequately be explained by reducing things to simple laws of nature. The world has to have been caused by something equally complex and beautiful (god). It's understandable where this misconception comes from. There are a lot of situations in our lives where we're actively maintaining some sort of order, and when we neglect things they break down. But as anyone who's ever spent any quality time with math can tell you, beauty readily sprouts from mundane beginnings all the time without the guiding hand of any conscious being, and we've found that this remarkable trend tends to carry over to our study of the universe (for more on that see the recent question on the math desk).
Here you seem to be making a similar argument about the impossibility of an interesting universe arising naturally without some "consciousness" guiding it, wrapped in (an incorrect reading of) the concept of entropy. Entropy doesn't measure how boring the universe is. With all the galaxies and stars and planets and life engaged in their beautiful dance, entropy is much higher than at the start of the universe when everything was a single boring (but maximally ordered) point. The magic is that bunch of deceptively simple laws (including the second law of thermodynamics) got us from there to here. Rckrone (talk) 04:04, 30 June 2011 (UTC)[reply]

Let's consider the life of "Adam", the first conscious being. In the interests of simplicity I will neglect to define whether Adam was man, mouse, or microbe; but his brain (or processing center of similar function) possessed the bare minimum requirement for what we define as a conscious observer. Thus there is one moment at which Adam is aware of consciousness, but ignorant of all else.

In that moment, Adam is, I would suppose, in a superposition of states, like Schroedinger's Cat. He might be Earthling or alien, with any number of possible biochemistries, in any number of universes with any number of laws - all existing as a mad superposition of states. But he stirs his limbs, and the state-vector collapses - as if struck by the divine spark, he becomes carbon-based life, with arms and legs; he takes on a defined form. Many others no doubt were possible. He opens his eyes and looks up to the darkened heavens - and at once, the state-vector of the skies collapses, and the stars, once a homogeneous smear of probability, take on their fixed and immovable positions. Wnt (talk) 05:53, 30 June 2011 (UTC)[reply]

Then he turns to the ominously fertile apparition at his side and simultaneously presents himself and enunciates the first palindrome: "Madam I'm Adam". Cuddlyable3 (talk) 12:00, 30 June 2011 (UTC)[reply]

String theory and time travel

Collapsing -- Wikipedia RefDesks are not a forum for presentation of original speculation
The following discussion has been closed. Please do not modify it.

Hi. Assume for the purposes of this question that String theory is correct. The theory makes the assumption of 11 dimensions, some of which are branes.

Suppose three of these dimensions are the spatial dimensions, and that time is the fourth dimension. Since strings are said to vibrate in the eleven dimensions, could a string in itself travel back and forwards in time, similar to quantum teleportation, except that these sub-quantum entities travel in the fourth rather than the 1-3 dimensions? If a particle collider were to harness enough energy to isolate strings, if they exist, let's say through the use of a Dyson sphere or some type of Higgs boson technology, could time travel or time-teleportation theoretically be achieved?

Since string theory supposes an infinite number of particles, far more than the current Standard Model, the mathematical singularity of a black hole would transcend some of these particles and reduce matter to infinite density at the string level. Since this requires energies capable of producing strings, could this allow time travel in the context of a wormhole via the tapping of the string energies? Could the black hole itself time travel to an earlier state in which it did not exist, thus evaporating into Hawking radiation and dissapating into dark energy?

Also, if time is one of the dimensions in which strings vibrate, how can the vibration occur without the flow of time? If strings were to travel in the fourth dimension, would the vibrations stop, or would this induce a parity in the time reversal symmetry present in any oscillation? Is this thus a form of supersymmetry breaking?

If strings vibrate based on an infinite particle number, could some high energy cause the vibrational frequency to change, and thus turning a neutron into a photon plus some energy, let's say? Or, perhaps create gravitons or other particles responsible for the other fundamental forces, so that the strings themselves impart force when enough energy is applied?

There are still many theories for light, including wave-particle duality and the neutrino theory. Apart from the existence of some disproven aether, would light simply be a form of converted energy, similar to the way that photons carrying energy cause electrons to jump to a higher level, releasing more energetic photons? Could strings facilitate this conversion, and generate new energy from particles?

At quantum levels, the observer effect becomes amplified, as seen in the quantum Zeno effect for the Heisenberg uncertainty principle. Is it conceivable that human observation is adjusting the vibrational frequencies of the strings, thus quantum-teleporting this string-information onto the later-observed states and momentum of the observed particles? Also, is it possible for an observer in the future, through this mechanism, to influence the quantum states of particles in the past, providing ostensible evidence of retrocausality and other retrospective mechanisms?

Responses welcome. Thanks. ~AH1 (discuss!) 00:59, 30 June 2011 (UTC)[reply]

Can I ask one question: can strings theoretically time travel? ~AH1 (discuss!) 01:51, 30 June 2011 (UTC)[reply]
So far as I know, all subatomic particles, real or proposed, exist in four dimensions, time being one. And most of them exist in a way which is in some sense symmetric over time (CPT symmetry). So when you ask if strings can time travel, well, you might have a string at one time, which exists in another time, earlier or later. Now can a change to a string later "cause" a change in the string earlier? That depends on the nature of "causation", which seems to me to be essentially religious, if not superstitious. Wnt (talk) 05:59, 30 June 2011 (UTC)[reply]

Request for RDS regulars to help with a teensy problem on another board...

Over at Wikipedia:In the news/Candidates, there's a consensus developing to have a blurb about a recently discovered quasar. However, no one has the science background to update or create a target article for the blurb. See the section "Brighest Object in the Galaxy found yet" We could use someone with astrophysics knowledge to pitch in to help work it out. Science is a heavily underrepresented topic in all areas of the main page, and ITN is no exception, so when a particularly good event comes along we don't want to miss the opportunity to use it. --Jayron32 01:56, 30 June 2011 (UTC)[reply]

Well, the person who could create that article is certainly not me, but I will make one correction: the object is clearly not in our galaxy. According to the article, it is the most distant object that has ever been observed. Do you think it would make sense simply to correct our quasar article, which currently says that all known quasars have redshifts between 0.056 and 6.5, to an upper limit of 7.085 (as reported in the Nature paper), and cite the paper? Would that be enough to justify a news item? Looie496 (talk) 02:11, 30 June 2011 (UTC)[reply]
Superlative objects (biggest, farthest, fastest, brightest, whatever) with good confirmation as to their superlative status are generally good topics for being stand-alone article subjects. Having Nature call you the brightest or farthest object known is a pretty solid aspect of notability, though again I (nor anyone currently working at WP:ITN/C) has the background to take on such a task. --Jayron32 02:24, 30 June 2011 (UTC)[reply]
For what it's worth, this Scientific American news piece tells the story in a pretty understandable way. Looie496 (talk) 02:26, 30 June 2011 (UTC)[reply]
Good stuff. There's an honest-to-goodness astronomer who has made a few comments, and indicated he may get around to working up an article in the next day or so. Of course, more help is more better. --Jayron32 02:29, 30 June 2011 (UTC)[reply]
Well, I just started one, ULAS J1120+0641. I'll say more at the ITN page. Looie496 (talk) 03:11, 30 June 2011 (UTC)[reply]

slow neutrinos

Hi, is there any FUNDAMENTAL reason why neutrinos could not be slowed down relative to us on Earth? If some type of exceedingly inelastic colission mechanism could be devised, couldn't scientists start collecting slow neutrinos? Once they are slow, wouldn't they stay slow? Conceivably the neutrino cross section would increase as they slowed down?Thanks, Rich Peterson24.7.28.186 (talk) 04:04, 30 June 2011 (UTC)[reply]

In principle they can be slowed down, but their cross-section would decrease. Have a look at formulas - the various cross sections are proportional to the energy or to the square of the energy. Icek (talk) 08:55, 30 June 2011 (UTC)[reply]

Plugging in and electrical use

I'm trying to save electricity, for the sake of both the environment and my bills. ;) So two questions:

(1) Do appliances such as fans with a plain adaptor-less power cord and no digital display (e.g. clock) use any electricity when they are turned off but still plugged into the electrical outlet? That is, would I save any energy by unplugging appliances even when they are turned off, or does being plugged in or not make no difference as long as the appliance is already turned off?

(2) What about devices that use AC adaptors? I find that even when I turn off the device, if I leave the adaptor plugged in, then the adaptor stays warm to the touch, suggesting it is still using electricity.

Also, if unplugging does indeed save power, are there any negative side effects to frequent unplugging?

SeekingAnswers (reply) 05:56, 30 June 2011 (UTC)[reply]

The phrase for those adaptors (and myriad other usages) is "power vampire". I still have no idea why corporate idiots decided to get rid of perfectly good 1 - 0 power switches that you could be confident about, and replaced them with things that consume more power while doing nothing than when active. There's just no plausible excuse. Wnt (talk) 06:03, 30 June 2011 (UTC)[reply]
Can you give an example of a "thing that consumes more power while doing nothing than when active? Cuddlyable3 (talk) 11:30, 30 June 2011 (UTC)[reply]
Consume more energy would be the right phrase. You buy energy not power. The CD-Player in the guest room only ocupied two weeks a year would be a clear point which would consume more during the 56 weeks in stand-by while working for 5 minutes during the guests are in the room. --Stone (talk) 11:44, 30 June 2011 (UTC)[reply]
Mechanical appliances do not normally use power when turned off, but there is an additional element of safety if they are switched off at two places. Appliances with "AC adaptors" (usually switch-mode power supplies) use a tiny amount of electricity when plugged in, but much less than a penny a day when not being used. I leave mine plugged in for convenience, but I live in a cold climate where the tiny amount of heat is beneficial. People who are fanatical about energy conservation always turn them off when not in use. Dbfirs 07:18, 30 June 2011 (UTC)[reply]

Expansion of the universe

Hello,

It is often said that the universe continues to expand, creating new space as it grows. However, this does not seem to fit in with the conservation of energy, since if new space is being created, that means new energy is being created in the form of vacuum energy. Am I missing something?

TIA. Leptictidium (mt) 08:16, 30 June 2011 (UTC)[reply]

This has come up before on the reference desk, but I don't have time to search at the moment. Here's a short version. First, it's problematic to say that "new space is created" in an expanding universe, since technically all of the space at a given time is equally new. Space doesn't persist the way matter does. Second, energy actually isn't conserved in cosmology (or GR in general), at least, not in any straightforward way. (This is actually an unsolved problem; quantum mechanics requires energy conservation, and that conflict is one of the reasons quantum gravity is hard.) -- BenRG (talk) 08:37, 30 June 2011 (UTC)[reply]

Regenerating electric energy in motor vehicles

in some motor vehicles electric energy can be stored through regenerative braking. so why cant we use this technology in electric cars to recharge the battery using this technology by an additional battery by providing a switchable control between the two? Since by Newton's Law "energy can neither be created nor be destroyed", i dont think dis is impossible..—Preceding unsigned comment added by 220.225.131.153 (talk) 12:32, 30 June 2011

This is already done in some hybrid carsZzubnik (talk) 12:38, 30 June 2011 (UTC)[reply]
Energy can not be created or destroyed, but if you convert some of your energy to heat, your car can't use it anymore, so for your purposes it might as well have been destroyed. Googlemeister (talk) 12:46, 30 June 2011 (UTC)[reply]

color

I don't fully understand why if a meterial absorbs,say,red light it emits blue light (or looks blue). and generally absorbing one color (wavelength,frequency,whatever)causes the object to emit the inverted color. and what is the definition of invert colors anyway? I searched "invert color" on wikipedia but found no satisfying results!thanks in advance.--Irrational number (talk) 12:56, 30 June 2011 (UTC)[reply]

The object doesn't emit the inverted colour, it reflects what's left of the incoming light (let's assume that's white) after it has absorbed some of it. Since now some wavelengths are missing from the reflected light, it takes on colour. For example, chlorophyll absorbs red light and blue light. The remainder is reflected, which is why tree leaves appear green. --Wrongfilter (talk) 13:04, 30 June 2011 (UTC)[reply]
Did you see Color#Color_of_objects ? Sean.hoyland - talk 13:10, 30 June 2011 (UTC)[reply]
Also, there are two confounding issues here:
  • 1) There are certain wavelengths of light which are absorbed, and others that are reflected. Sometimes, objects do "re-emit" light (see fluorescence and phosphorescence) but reflected light and emitted light are distinctly different and can be readily identified in most situations. For normal (non fluorescing) objects, reflected light determines what wavelengths reach our eyes.
  • 2) Human color perception determines how the incoming wavelengths of light strike our eye and are processed by our brain to produce a distinct color in our eyes. It is not always obvious; for example there are actually two different ways that yellow can be produced for us: a single wavelength of light in the yellow range can look yellow, but light composed of a mixture of wavelengths of light from the red and green ranges, with no actual wavelengths from the yellow range will still look yellow to us. That sort of thing is why we can create a full pallete of colors from a limited number of pigments, see RGB and CMYK for some more info on that. --Jayron32 13:11, 30 June 2011 (UTC)[reply]
(Edit Conflicts) Despite having studied some Color theory in connection with printing, I've never encountered the term "invert[ed] color. You may however find further enlightenment at the articles Complementary color, Primary colors and Secondary colors. {The poster formerly known as 87.81.230.195} 90.197.66.166 (talk) 13:14, 30 June 2011 (UTC)[reply]

What is this little white bug?

I was photographing insects in Colonial Williamsburg, Virginia this past weekend when I noticed this little one. I have never seen anything like it, I'm guessing it is a nymph of some sort? It looks like a miniature, white dinosaur.

http://keeganm.com/gallery/2011_06_26/images/large/DSC_3345.jpg

Thanks! Keegstr (talk) 13:33, 30 June 2011 (UTC)[reply]

An Aphid of some sort? There are dozens and dozens of species of aphids, perhaps one of them? --Jayron32 14:28, 30 June 2011 (UTC)[reply]
Cool photo! It is somewhat uncommon to see a singular wingless aphid (because they rapidly form clonal aggregations), and aphids are usually seen with their stylets inserted into stems or leaves. What kind of plant was it on? Did it run away when disturbed, or stay put? I think you are correct that it is a nymph. I'll keep looking... SemanticMantis (talk) 14:58, 30 June 2011 (UTC)[reply]
Awesome. If you think it would be beneficial to post the image to a specific wiki article, I will gladly do so if you can tell me what to title it and where to put it. Here is the full photo (above is a 100% crop) I am unsure of the flower type. http://keeganm.com/tmp/DSC_3345-2.jpg Keegstr (talk) 15:08, 30 June 2011 (UTC)[reply]